Sie sind auf Seite 1von 115

SYLLABUS

ACCOUNTING FOR
MANAGERS
MBA–1st SEMESTER, M.D.U., ROHTAK
External Marks : 70 Internal Marks : 30
Time : 3 hrs.

UNIT-I
Financial Accounting-concept, importance and scope,
accounting
principles, journal, ledger, trial balance, depreciation (straight line
and
diminishing balance methodology), preparation of final accounts
with
adjustments
.

UNIT-II
Ratio analysis, fund flow analysis, cash flow
analysis.

UNIT-III
Management accounting-concept, need, importance and scope;
cost
ac c ountin g- me aning, importanc e, me thods, tec hniq ue s a
nd
classification of costs, inventory
valuation.

UNIT-IV
Budgetary control-meaning, need, objectives, essentials of
budgeting,
different types of budgets; standard costing and variance
analysis
(materials, labour); marginal costing and its application in
managerial
decision
making.

143

footer
ACCOUNTING FOR
MANAGERS
MBA 1st Semester
(DDE)

UNIT – I

Q. Define Accounting. Explain its


Nature.
Ans. Accounting
Accountingis often
:- called the language of business.
The function of any language is to communicate. Accounting
basic
communicates
the results of the business to the users of accounting information to
enable
them to make effective decisions. To communicate information,
accounting
follows a systematic process of recording, classifying and summarizing
of
numerous business transactions resulting in creation of financial
statements.
The two most important financial statements
are :–
(i) Trading, Profit & Loss
Account.
(ii) Balance
Sheet.
Definition of :–
Accounting
According to American Institute of Certified Public
Accountants:–
“Accounting is the art of recording, classifying and summarizing in a
significant
manner and in terms of money, transactions and events which are, in
part
atleast, of a financial character, and interpreting the results
thereof”.
According to R.N. Anthony :–
“Nearly every business enterprise has accounting system. It is a means
of
collecting, summarizing, analyzing and reporting in monetary
terms,
informations about
business”.
Feature or Characteristics or Nature of :–
Accounting
(1) Recording of Financial Transactions Only those
only :–and events are recorded in accounting which can be expressed transactions
in terms
of
money. Those transactions which cannot be expressed in terms of
money
are not recorded in accounting like the value of human resource, strike
by
employees, and change in managerial policies
etc.
(2) Accounting is the art of recording of business
Recording :– to sometransactions
according specified rules. In a small business where number
of
transactions is quite small, all transactions are first of all recorded
in a
144

footer
ACCOUNTING FOR
MANAGERS
book called “Journal”. But in a big business where the number
of
transactions is quite large, the Journal is further sub-divided into
various
subsidiary books such
as:- (i) Cash Book

(ii) Purchase
Book
(iii) Sales
Book
(iv) Purchase Return
Book
(v) Sales Return
The numberBook.
of subsidiary books to be maintained depends on the size
and
nature of the
business.
(3) After recording the transactions in journal or
Classifying
books,:–the transactions
subsidiary
are classified. Classification is the process
of
grouping the transactions of one nature at one place, in a
separate
account. The books in which various accounts are opened is
called
“Ledger”
.
(4) Summarising involves the balancing of Ledger
Summarising
and the :–preparation accounts
of Trial Balance with the help of such
Balances.
Financial Statements are prepared with the help of trial balance.
Financial
statements are
includes:-
(i) Trading, Profit & Loss
Account
(ii) Balance
Sheet.
(5) Interpretation of the In accounting the results of business
Results :–
presented are
in such a manner that the parties interested in the
business
such as proprietors, managers banks, creditors etc. can have
full
information about the profitability and the financial position of
the
business
(6) . It refers to transmission of summarized
Communicating
interpreted :–
information to aand
variety of users. The users
are:- (i)
Creditors
(ii)
Investors
(iii)
Lenders
(iv)
Government
(v)
Proprietors
(vi)
Management
(vii) Banks
Q. Defineetc. Accounting. Also explain its
Importance.
Ans. Accounting
Accountingis often
:– called the language of business.
The function of any language is to communicate. Accounting
basic
communicates
145

footer
the results of the business to the users of accounting information to
enable
them to make effective decisions. To communicate information,
accounting
follows a systematic process of recording, classifying and summarizing
of
numerous business transactions resulting in creation of financial
statements.
The two most important financial statements
are:- (i) Trading, Profit & Loss
Account.
(ii) Balance
Sheet.
Definition of
Accounting
According :–to American Institute of Certified Public
Accountants :– is the art of recording, classifying and summarizing in
“Accounting
a
significant manner and in terms of money, transactions and events which
are,
in part atleast, of a financial character, and interpreting the results
thereof”.
Importance of
Accounting :–
(1) Helpful in Management of Management needs a lot
Business :–
information of
for the efficient running of the business. All such
information
is provided by the accounting which helps the management in
following:
the
-
(i) Helpful in Management would like to know whether
Planningsales:–are increasing or decreasing
the and also the speed of increase
in
the cost of production. All such information is provided by
the
accounting, which helps the management in estimating the
future
sales and expenses. It also helps them to estimate the cash
and cash disbursements during the next accounting
receipts
period.
(ii) Helpful in Decision- At times, the Management has
Making :– a number of decisions. Accounting
take to provides all the
informations
required for making such
decisions.
(iii) Helpful in Management would like to see that the
incurred
Controlling :– is reasonable and cost that no department is
overspending.
Accounting provides information to the management in this
regard.
(2) Provides Complete and Systematic Business
Record :– grown in size and complexity and it is not possible to remember
have transactions
and every transaction. Accounting keeps a prompt and systematic
each
record
of all the transactions and summarizes them in order to provide a
true
picture of the activities of the business
entity.
(3) Information regarding Profit or Accounting reports the
result of business activities of an accounting period.netFor this
Loss :–
purpose
Trading and Profit & Loss Account of the business is prepared at the end
of
each accounting period. All the items relating to purchase, sales,
expenses
and revenues (Income) of the business are recorded in Trading, Profit
&
Loss Account.

146

footer
ACCOUNTING FOR
MANAGERS
If Revenues >Expenses-—————————————
Profit
If Revenues< Expenses-—————————————
Loss
(4) Information Regarding Financial For a
Position :– ascertaining profit or loss of the business is not sufficient.
merely businessman,
The
businessman must also know the financial health of the business. For
this
purpose a statement called Balance Sheet is prepared which shows
assets on the one hand and the liabilities and capital on the other
the
hand.
Balance Sheet describe the
following :–
(i) How much the business has to recover from
Debtors?
(ii) How much the business has to pay to
Creditors?
(iii) How much the business has in the form
of (a) Cash-in-hand (b) Cash at
Bank
(c) Closing Stock (d) Fixed
Assets.
(5) Enables Comparative By keeping a systematic
Studyaccounting
:– helps the owners to compare one record
year’s costs, expenses,
sales
and profit etc. with those of other years. Such a comparison provides
the
useful information on the basis of which important decisions can be
more
taken
judiciously.
(6) Provide Informations to Various Another main objectives
Parties :–
accounting is to communicate the accounting information of to
various
users
like: (i)
Creditors
(ii)
Investors
(iii)
Lenders
(iv)
Government
(v)
Proprietors
(vi)
(vii)
Management Banks
etc.
(7) To Know the Liquidity Another objective of accounting is
Position :–
provide to For this purpose it
information about liquidity position.
prepares
a Cash Flow Statement. It depicts inflows and outflows of cash
from
operating, investing and financing
(8) activities.
To File Tax One of the main objectives of accounting is
Returns :– bases for filing tax returns
provide to relating to income tax, sales tax,
value
added tax, service tax,
etc.
(9) Facilitates Sale of If a business entity is being sold,
Business :–
accounting information can be utilized the
to determine the proper
purchase
price
(10) . Helpful in Raising Accounting information is of great help
Loansraising
:– whilefinancial institutions. Such
loans from banks or other
institutions

147

footer
before sanctioning loan screen various financial statements of the
firm
such as final accounts, fund flow statement, cash flow statement
etc.
(11) Helpful in Prevention and Detection of Errors and
Frauds.
Q. Define Accounting. Also explain its
Scope.
Ans. Accounting
Accountingis often
:– called the language of business.
The
basic function of any language is to communicate. Accounting
communicates
the results of the business to the users of accounting information to
enable
them to make effective decisions. To communicate information,
accounting
follows a systematic process of recording, classifying and summarizing
numerous business transactions resulting in creation of financial
of
statements.
The two most important financial statements
are:- (i) Trading, Profit & Loss
Account.
(ii) Balance
Sheet.
Definition of
Accounting
According :–to American Institute of Certified Public
Accountants
“Accounting :–is the art of recording, classifying and summarizing in a
significant
manner and in terms of money, transactions and events which are, in
part
atleast, of a financial character, and interpreting the results
thereof”.
Scope of In order to appreciate the exact nature and scope
Accounting :–we must understand
accounting, of the following aspects of
accounting:
(1) Economic Accounting records only economic events.
Eventseconomic
:– Anwhich can be measured and expressed
event is a transaction
in
terms of
money.
(2) It means determining what transactions are to
Identification
recorded.:–It involves observing
be events and selecting those events that
are
of financial character and relate to the
(3) organization. It means quantification of business transactions
Measurement
financial :–terms by using
into monetary
units.
(4) Accounting is the art of recording of business
according
Recording :– to some specified rules. In a small business where number
transactions
of
transactions is quite small, all transactions are first of all recorded
in a called “Journal”. But in a big business where the number
book
of
transactions is quite large, the Journal is further sub-divided into
various
subsidiary books such
as:-
Ø Cash Book
Ø Purchase Book
Ø Sales Book
Ø Purchase Return
Ø Book
Sales Return
Book.
148

footer
ACCOUNTING FOR
MANAGERS
The number of subsidiary books to be maintained depends on the size
nature
and of the
business.
(5) After recording the transactions in journal or
Classification :–transactionssubsidiary
books, the are classified. Classification is the process
of
grouping the transactions of one nature at one place, in a
separate
account. The books in which various accounts are opened is
called
“Ledger”
(6) . Summarising involves the balancing of Ledger
Summarising
and the :–preparation accounts
of Trial Balance with the help of such
Balances.
Financial Statements are prepared with the help of trial balance.
Financial
statements are
includes:-
(i) Trading, Profit & Loss
Account
(ii) Balance
Sheet.
(7) It refers to transmission of summarized
interpreted :–
Communication information to aand
variety of users. The users
are:-
(i)
Creditors
(ii)
Investors
(iii)
Lenders
(iv)
Government
(v)
Proprietors
(vi)
Management
(vii) Banks
etc.
(8) Interpretation of the In accounting the results of
Results
are:–presented in such a manner that the business
parties interested in the
business
such as proprietors, managers banks, creditors etc. can have
full
information about the profitability and the financial position of
the
business
.
Q. Define Accounting. Explain its Objectives Or Functions and
Branches
Or Types.
Ans. Accounting
Accountingis often
:– called the language of business.
The
basic function of any language is to communicate. Accounting
communicates
the results of the business to the users of accounting information to
enable
them to make effective decisions. To communicate information,
accounting
follows a systematic process of recording, classifying and summarizing
of
numerous business transactions resulting in creation of financial
statements.
The two most important financial statements
are:- (iii) Trading, Profit & Loss
Account.
(iv) Balance
Sheet.
Definition of
Accounting :–to American
According Institute of Certified Public
Accountants :–
149

footer
“Accounting is the art of recording, classifying and summarizing in a
significant
manner and in terms of money, transactions and events which are, in
part
atleast, of a financial character, and interpreting the results
thereof”.
Objectives or Functions of The following are the
Accounting:- functions or utility of
objectives, main
accounting:-
(1) To keep a Systematic record of business The main
transactions :– of accounting is to maintain complete record of
objective
business
transactions according to some specified rules. For this purpose all
the
business transactions are first of all recorded in Journal or
Subsidiary
Books and then posted into
Ledger.
(2) To Calculation Profit or The second main objective of
Loss :–is to calculate the net profit earned oraccounting
loss suffered during a
period. For this purpose Trading and Profit & Loss Account of the
particular
business
is prepared at the end of each accounting period. All the items relating
to
purchase, sales, expenses and revenues (Income) of the business
are
recorded in Trading, Profit & Loss
Account.
If Revenues >Expenses -—————————————
Profit
If Revenues< Expenses-—————————————
(3) To know Loss
the exact reasons leading to net profit or net
loss.
(4) To Know the Financial Position of the For a
business :– ascertaining profit or loss of the business is not sufficient.businessman,
merely
businessman
The must also know the financial health of the business. For
this
purpose a statement called Balance Sheet is prepared which shows
the
assets on the one hand and the liabilities and capital on the other
hand.
(5) To ascertain the progress of the business from year to
year.
(6) To prevent and detect errors and
frauds.
(7) To Provide Informations to Various Another main
Parties
of:–accounting is to communicate the accounting information
objectives
to
various
users
(8) .To Know the Liquidity Another objective of accounting is
Position :–
provide to For this purpose it
information about liquidity position.
prepares
a Cash Flow Statement. It depicts inflows and outflows of cash
from
operating, investing and financing
activities.
(9) To File Tax One of the main objectives of accounting is
Returns :– bases for filing tax returns
provide to relating to income tax, sales tax,
value
added tax, service tax,
etc.
Branches OR Types of :– Branches of accounting
Accounting are :–
(1) Financial It covers the preparation and interpretation
Accounting :– of
150

footer
ACCOUNTING FOR
MANAGERS
financial statements and communication to the users of accounts.
The
final step of financial accounting is the preparation of Trading and Profit
&
Loss Account and the Balance
Sheet.
(2) Management The main purpose of
Accounting :– is to present the accounting information
Accounting Management in such a way as
to
assist the management in planning and controlling the operations of
abusiness. The management accountant uses various techniques
and
concepts to make the accounting data more useful for managerial
decision
making.
(3) Tax Accounting :– The branch of accounting which is used for
tax
purpose is called tax accounting. Income Tax and Sale Tax are
computed
on the basis of this
accounting.
(4) Cost The main purpose of cost accounting is to
Accounting :– cost and per unit calculate
the total cost of goods produced and services rendered
by
a business. It also estimates the cost in advance and helps
the
management in exercising strict control over
cost.
(5) Social Responsibility The society provides
Accounting :–
infrastructure the
and the facilities without which business cannot operate
at
all. Hence the business also has a responsibility to the society. There
is a
growing demand for reports on activities which reflect the contribution
of
an enterprise to the society. Social responsibility accounting is
the
process of identifying, measuring and communicating the contribution
of
a business to the society. In social responsibility accounting
techniques
have been developed for measuring the cost of these contribution and
the
benefits to the
society.
Q. What do you mean by Accounting Principles or (GAAP)? Explain
and illustrate
fully.
Ans. The
Accounting
accounting statements are:–needed
by Principle
various parties who have interest in the business, namely,
proprietors,
investors, creditors, government and many other. Accounting
statements
disclose the profitability and solvency of the business to various parties. It
is
therefore, necessary that such statements should be prepared according
to
some standard language and set rules. These rules are usually called
‘General
Accepted Accounting Principles’
(GAAP).
Kinds of Accounting :– Accounting principles are described
Principles
various terms such as assumptions, conventions, by concepts,
doctrine,
postulate etc. These principles can be classified mainly into two
categories:-
(A) Accounting Concepts or
Assumptions
(B) Accounting
Conventions.
151

footer
Kinds of Accounting
Principles

Accounting Accounting
Concepts
or Conventions
Assumptions
Business Entity Convention of Full
Concept Disclosure
Money Measurement Convention of
Concept Consistency
Going Concern Concept Convention of
Conservatism
Accounting Period Convention of
Concept Materiality
Historical Cost
Concept
Dual Aspect
Concept
Revenue Recognition
Concept
Matching
Concept
Accrual
Concept
Objectivity
Concept

(A) Accounting Concepts or Accounting concepts


Assumptions :–
the assumptions define of a
on the basis of which financial statements
business
entity are prepared. The word concept means idea or notion, which
has
universal application. These accounting concepts provide a foundation
for
accounting process. No enterprise can prepare its financial
statements
without considering these basic concepts or assumptions. These
concepts
guide how transactions should be recorded and reported. Following
be treated as basic concepts or
may
assumptions :–

(1) The Business Entity Entity concept states that


Concept :–
enterprise is a separate identity apart frombusiness
its owner. Accountants
should
treat a business as distinct from its owner. Business transactions
are
recorded in the business books of accounts and owner’s transactions
in
his personal books of accounts. Business unit should have a
separate set of books and we have to record business transactions
completely
from
firm’s point of view and not from the point of view of the
proprietor.
Example :–

152

footer
ACCOUNTING FOR
MANAGERS
(i) The proprietor is treated as a creditor of the business to the extent
of capital invested by him in the business. The capital is treated as
aliability of the firm because it is assumed that the firm has
borrowed
funds from its own proprietors instead of borrowing from
outside
parties. It is for the reason that we also allow interest on capital
treat it as an expense of the
and
business. the amount withdrawn by the proprietor from the
(ii) Similarly,
business
for his personal use is treated as his
drawings.
(iii) The proprietor’s house, his personal investment in securities,
his personal car and personal income and expenditure are kept
separate
from the accounts of the business
(iv) Ifentity.
the proprietor has some other business entity doing
another business, the records of that business should also be kept
The conceptseparate.
of separate entity is applicable to all forms of
business
organizations, i.e. sole proprietorship, partnership or a
company.
(2) Money Measurement As per this concept, only
Concept :–
transactions, which can be measured in terms thoseof money are
recorded.
Transactions, even if, they affect the results of the business materially,
are
not recorded if they are not convertible in monetary terms.
Transactions
and events that cannot be expressed in terms of money are not recorded
the business books. For example, accounting does not record a
in
quarrel
between the production manager and sales manager; it does not
report
that a strike is beginning and it does not reveal that a competitor
has
placed a better product in the market. These facts or happenings
cannot
be expressed in money terms and thus are not recorded in the
books.
Example :– A business on a particular day has 5000 Kilograms of
raw 100 Chairs and 20 Fans. All these things cannot
materials, 5 Machines,
be
added up unless expressed in terms of money. In order to make a record
of
these items, these will have to be expressed in monetary terms such as
Raw
Materials Rs. 25000, Machines Rs. 200000, Chairs Rs. 5000 and Fans
8000.
Rs. As such, to make accounting records relevant, simple,
understandable
and homogeneous, they are expressed in a common unit of measurement
i.e.,
money.
(3) Going Concern As per this concept it is assumed that
Concept :–
business will continue to exist for athe
long period in the future.
The
transactions are recorded in the books of the business on the
assumption
that it is a continuing
enterprise.
Example :–
(i) It is on this concept that we record fixed assets at their original
cost and depreciation is charged on these assets without reference to
their
market
value.
153

footer
(ii) It is also because of the going concern concept that outside
partiesenter into long-term contracts with the enterprise, gives loans
and
purchase the debentures and shares of the
enterprise.
(iii) Another example of this concept is that Prepaid Expenses,
whichhave no realizable value are shown as assets in the balance
sheet,
because the benefits of such expenses will be received in
(4) future.
Accounting Period According to this concept
Concept :– be prepared after every period & notaccounts
should at the end of the life of
the
entity. Usually this period is one calendar year i.e. 1 Jan to 31st s t

December
or from 1 April to 31 March. According to Amended Income Tax Law,
stst

a
business has compulsorily to adopt financial year beginning on 1 st

April
and ending on 31 March. Apart from this, companies whose shares
st

are on the stock exchange are required to publish quarterly results


listed
to
depict the profitability and financial position at the end of three
months
period.
(5) Historical Cost Concept or Cost According to this
Concept :– is ordinarily recorded in the books of accounts at the
an asset concept,
price
at
which it was purchased or acquired. This cost becomes the basis of
all
subsequent accounting for the asset. Since the acquisition cost relates
to
the past, it is referred to as historical cost. This cost is the basis
of
valuation of the assets in the financial
statements.
Example :– If a business purchases a building for Rs. 500000, it would
recorded in thebe books at this figure. Subsequent increase or decrease
in
the market value of the building would not be recorded in the books
of
accounts
.
Benefits :
–(i) It is highly objective and free from
bias.
(ii) Market values of assets are difficult to be
determined.
(iii) Market values of the assets may change from time to time and it
will be extremely difficult to keep track of up and down of the
market
price
.
Limitations :

(i) Assets for which nothing is paid will not be recorded. Thus
a favourable location, brand name and reputation of the
business,
knowledge and technological skill built inside the enterprise
will
remain unrecorded though these are valuable
(ii) assets.
Historical cost-based accounts may lose
comparability.
(iii) Many assets do not have acquisition
cost.
(iv) During periods of inflation, the figure of net profit disclosed by
profit and loss account will be seriously distorted because
depreciation
154

footer
ACCOUNTING FOR
MANAGERS
based on historical costs will be charged against revenues at
current
prices
.
(v) Information based upon historical cost may not be useful
to management, investors, creditors
etc.
(6) Dual Aspect According to this concept, every
Concept :–
transaction is recorded as having business
a dual aspect. In other words,
every
transaction affects atleast two accounts. If one account is debited,
any
other account must be credited. The system of recording
transactions
based on this concept is called as ‘Double Entry System’. It is because
of
this principle that two sides of the Balance Sheet are always are equal
and
the following accounting equation will always hold good at any point
time:
of
-
Assets = Liabilities +
Capital
OR
Capital = Assets -
Liabilities
Example :– X commences business with Rs. 5 Lacs in cash and takes
a from the bank, and these 6 Lacs are used in buying
loan of Rs. 1 Lac
assets, say, plant & machinery. The equation will be as
some
follows: Assets = Liabilities +
Capital
Rs. 6 Lacs = Rs. 1 Lac + Rs. 5
Lacs
(7) Revenue Recognition (Realisation) Revenue means
Concept :– which is added to the capital as a result of business
amount the
operations.
Revenue is earned by sale of goods or by providing a service. Concept
of
revenue recognition determines the time or the particular period in
which
the revenue is realized. Revenue is deemed to be realized when the title
or
ownership of the goods has been transferred to the purchaser and
when
he has legally become liable to pay the amount. It should be
remembered
that revenue recognition is not related with the receipt of
cash.
Example :– For example, if a firm gets an order of goods on 1 st

January,
supplies the goods on 20 January and receives the cash on 1 April,
th st

the
revenue will be deemed to have been earned on 20 January, as th

the
ownership of goods was transferred on that
day.
(8) Matching This concept is very important for
Concept :–
determination correct to this concept, all expense
of net profit. According
matched with the revenue of that period should only be taken
are
into
consideration. This principle is based on accrual concept as it
considers
the occurrence of expenses and income and do not concentrate on
actual
inflow or outflow of cash. This principle helps us in finding Net profit
or
Loss. Following points must be considered while matching costs
with
revenue
:
155

footer
(i) When an item of revenue is included in the profit and loss account,
all expenses incurred on it, whether paid or not, should be show
as
expenses in the profit and loss
account.
(ii) When some expenses, say insurance premium is paid partly for
the next year also, the part relating to next year will be shown as
an
expense only next year and no this
year.
(iii) Similarly, income receivable must be added in revenues and
incomes received in advance must be deducted from
revenues.
(9) Accrual In accounting, accrual basis is used for
transactions.
Concept :– It provides more appropriate information about
recording
the
performance of business enterprise as compared to cash basis.
Accrual
concept applies equally to revenues and expenses. In accrual
revenue is recorded when sales are made whether cash is received or
concept
not.
Similarly, according to this concept, expenses are recorded in
the
accounting period in which they assist in earning the revenues
whether
the cash is paid for them or
(10) not. Objectivity This concept requires that
Concept :–
transaction should be recorded in anaccounting
objective manner, free from
personal bias of either management or the accountant who prepares
the
the
accounts
(B) . Accounting An accounting convention may be defined
Conventions
a custom:– or generally accepted practice
as which is adopted either by
general
agreement or common consent among accountants .
Accounting
conventions differ from concept in respect to the
following:
(i) Accounting concepts are established by law while
accounting
conventions are guidelines based upon general
agreement.
(ii) There is no role of personal judgment or individual bias in
the adoption of accounting concepts whereas they may play a crucial
role
in following accounting
conventions.
(iii) There is uniform adoption of accounting concepts in
different
enterprise while it may not be so in case of accounting
Following conventions.
are the main accounting
conventions
(1) :–
Conventions of Full This principle requires that
Disclosure :–
significant information relating to the economic all affairs of the
should be completely disclosed. The principle is so important that
enterprise
the
companies Act makes ample provisions for the disclosure of
essential
information in the financial statements of a company. The proforma
and
contents of Balance Sheet and Profit and Loss Account are prescribed
Companies Act. Various items or facts which do not find place
by
in
accounting statements are shown in the Balance Sheet by way
of
footnotes. Such
as :
156

footer
ACCOUNTING FOR
MANAGERS
(i) Contingent
Liabilities.
(ii) If there is a change in the method of valuation of stock, or
for providing depreciation or in making provision for doubtful debts,
it
should be disclosed in the Balance Sheet by way of a
footnote.
(iii) Market value of investments should be given by way of a
footnote.
(2) Convention of According to this principle,
principles
Consistency :– and methods should remain consistent from one year
accounting
to
another. These should not be changed from year to year. If a firm
adopts
different accounting principles in two accounting periods, the profits
of
current period will not be comparable with the profits of the
preceding
period.
(3) Convention of According to this principle,
Conservatism :– losses should be recorded in the books
anticipated all of accounts, but
all
anticipated gains should be ignored. In other words, conservatism is
the
policy of playing safe. When there are many alternative values of an
asset,
an accountant should choose the method which leads to the lesser
value.
Examples of the application of the principle of
conservatism :– closing stock – ‘cost or market price’ whichever is
(i) Valuation of
less.
(ii) Provision for Doubtful Debts on
Debtors.
(iii) Joint Life Policies are recorded at Surrender
Values.
Effects of Principle of
Conservatism :– account will disclose lower profits in comparison to
(i) Profit & Loss
the actual
profits.sheet will discloses understatement of assets
(ii) Balance
and overstatement of liabilities in comparison to the actual
values.
(4) Convention of This convention is an exception to
Materiality :– of full disclosure. According
convention theto this convention, all the
items
having significant economic effect should be disclosed in
financial
statements and any insignificant item which will only increase the work
of
the accountant should not be disclosed in the financial statements.
It
should be noted that what is material for one concern may be
immaterial
for another. Thus, the accountant should judge the important of
each
transaction to determine its
materiality.
Q. Give Classification Of Accounts. What are the Rules of
Journalising?
Ans. Classification
Classification of Accounts
of :–
are: Accounts Classification of Accounts

Personal Accounts Impersonal Accounts

Real Accounts Nominal Accounts

157

footer
1. Personal The accounts which relate to an individual,
Accounts :– or an institution arefirm,
company called personal accounts. Account
of
Mohan, Account of D.C.M. Limited, Capital Account of proprietor, etc.
are
the examples of Personal Accounts. This account is further classified
into
three
categories:-
(i) It Natural
relates to transactions
Personal
of
humanbeings
Accounts :– Ram, Rita,
like
etc.
(ii Artificial Personal These accounts do not have
) Account :–
physical existence as human beings buta they work as
personal
accounts. For example: Government, Companies (private or
limited),
Clubs, Co-operative Societies
etc.
(iii Representative Personal These are not in the name
) Accounts
any person :– or organization but are represented of as personal
account.
For Example: Outstanding liability account or prepaid
account,
capital account, drawings
account.
Golden Rule of Personal
Account
Debit :– the
Receiver
Credit the
Giver
2. Impersonal Accounts which are not personal such
Account :–
machinery account, cash account, as rent account etc. These can be
further
sub-divided as
follows :–
(i) Accounts
Real Account
which relate
: to assets of the firm but
not –debt. For example accounts regarding Land, Building,
Investment,
Fixed Deposits etc., are real accounts Cash-in-hand and Cash
at
Bank are also
real.
Golden Rule of Real
Account :–
Debit what comes
in.
Credit what goes
out.
(ii) Accounts
Nominal which relates to expenses,
losses,Account :–
gains, revenue etc. like salary account, interest paid
account,
commission received
account.
Golden Rule of Nominal
Account
Debit :–all expense &
Losses.
Credit all Incomes &
Gains.
Q. Define Accounting Cycle OR Process of
Accounting.
Ans. An
Accounting
accounting cycle is a complete
:– sequence
of Cycleprocedures which are repeated in the same order during
accounting
each
accounting period. The accounting cycle may be shown as
below:-
158

footer
ACCOUNTING FOR
MANAGERS
Transactions

Books of Original Entry:


1. Cash Book
2. Purchase Book
Trading, Profit & 3. Sales Book
Loss A/c and Journal 4. Purchase Return Book
Balance Sheet
5. Bills Receivable Book
6. Bills Payable Book
7. Journal Proper

Trial Balance Ledger

Diagram : Accounting
Cycle

(1) Identification of Accounting deals with


Transaction :– which are monetary in nature. In other
transactions business
words,
the
transactions which cannot be measured and expressed in terms of
cannot
money be recorded in
accounting.
(2) Journal :– Journal is one of the basic book of original entry in
transactions arewhich
recorded in a chronological (day-to-day) order
according
to the principles of double entry system. When the size of business is
asmall one, it may be possible to record all transactions in the journal
but
when the size of the business grows and the number of transactions is
very
large journal is sub-divided into a number of books called
subsidiary
Books.
There are five columns in journal which
are:- PROFORMA OF JOURNAL

Date Particulars L.F. Amount Dr. Amount


Cr.
(1) (2) (3) (4)
(5)

(i) In Date
the first
:– column, date of transaction is entered. The
year and month is written only once, till they
change.
(ii) Each
Particulars
transactions
: affects two accounts out of
which–one account is debited and other account is
credited.
159

footer
(iii) All
Ledger
entries
Folio
from
or L.F.
the :–
journal are later
postedinto the ledger accounts. The page number of the ledger
account
where the posting has been made from the journal is recorded in
the
L.F. column of the journal.
(iv) InAmount
the fourth
Dr. :–
column, the amount of the account
being debited is
written.
(v) InAmount
the fifthCr.
column,
:– the amount of the account
being credited is
written.
(3) Business transactions are first recorded in journal
Subsidiary
Ledger :– books.
or The next step is to transfer the entries to
respective
accounts in ledger. This process is called
ledger
Dr. Cr.

Date Particulars J.F. Amount Date Particulars J.F.


Amount

Each ledger account is divided into two equal parts. The left-hand side
is
known as the debit side and the right-hand side as the credit
side.
As shown above, there are four columns on each side of an
account:-
(i) The Date
date
:– of the transaction is recorded in this
column.
(ii) EachParticulars
transaction
: affects two
accounts.
(iii) In–Journal
this column
Foliopage
or J.F.
number
:– of the journal
or subsidiary book from which the particular entry is transferred,
is
entered
.
(iv) The
Amount
amount
:– is entered in this
(4) column. Trial When posting of all the transactions into ledger
Balance :–
completed is are balanced off, it becomes necessary
and the accounts
to
check the arithmetical accuracy of the accounting work. For this
purpose,
the balance of each and every account in the ledger is put on a list. The
list
so prepared is called a trial
balance.
PROFORMA OF TRIAL BALANCE

Name of the Accounts L.F. Dr. Balances Cr.


Balances

Features of a Trial
Balance
(i) :–
It is a list of balances of all ledger accounts and the cash
book

160

footer
ACCOUNTING FOR
MANAGERS
(ii) It is just a statement and not an
account.
(iii) It is neither a part of double entry system, nor does it appear in
the actual books of accounts. It is just a working
(iv) It paper.
can be prepared at any time during the accounting period, say,
at the end of every month, every quarter, every half year or every
(v) It year.
is always prepared on a particular date and not for a
particular
period.
(vi) It is prepared to check the arithmetical accuracy of the
ledgeraccounts
(vii) If. the books are arithmetically accurate, the total of all debit
balancesof a trial balance will be equal to the total of all credit
Objectives balances.
of Preparing Trial
Balance(i) :–To ascertain the arithmetical accuracy of the ledger
accounts.
(ii) To help in locating
errors
(iii) To obtain a summary of the ledger
accounts
(iv) To help in the preparation of final
accounts.
(5) Trading, Profit & Loss Account And Balance :– After
Sheet the accuracy of the book of accounts through preparation of having
checked
Trial
Balance, businessman wants to ascertain the profit earned or
loss
suffered during the year and also the financial position of his business
at
the end of the year. For this purpose he prepares ‘Final Accounts’
which
are also termed as” Financial Statements. These include the
following:-
Ø Trading Account.
Ø Profit and Loss
Ø Account.
Balance
Sheet.
Q. Write a Short Note On Double Entry
System.
Ans. Double
According to Double
Entry Entry :– System,
every System
transaction has two fold-aspects- debit and credit and both the aspects are
to
be recorded in the books of accounts. We may define the Double Entry
System
as the system which records both the aspects of transactions. This
principle
proves accounting equation i.e. both sides of Balance Sheet always
equal.
Assets = Liabilities +
Advantages ofCapital
Double Entry This system affords the
System :–
mentioned under
advantages :–
(1) Scientific
System
(2) Complete Record of Every
Transaction
(3) Preparation of Trial
Balance
(4) Preparation of Trading & Profit & Loss
A/c
(5) Knowledge of financial position of the
Business
(6) Knowledge of Various
Informations.
161

footer
(7) Comparative
Study
(8) Lesser possibility of
Fraud.
(9) Help management in decision
making.
(10) Legal
Approval
(11) Suitable for All types of
Businessmen.
Q. Define Depreciation. What are the Causes & Methods
of Depreciation
?Depreciatio
Ans. In every business there
:– are certain assets of a
fixed n that are needed for the conduct of business operations. Some
nature
examples
of such assets are Building, Plant & Machinery, Motor Viechles,
Furniture,
office Equipments etc. These assets have a definite span of life after the
expiry
of which the assets will lose their usefulness for the business operations. Fall
in
the value & utility of such assets due to their constant use and expiry of time
is
termed as
depreciation.
Definition of :–
Depreciation
According to William
Pickles“Depreciation may be defined as the permanent and
continuing
diminution in the quality, quantity or the vale of an
asset”.
Features of :–
Depreciation
1. Depreciation is decline in the value of fixed assets (except
Land)
2. Such fall is of a permanent
nature.
3. Depreciation is a continuous process because value of assets
will decline by their constant
use.
4. Depreciation decreases only the book value of the asset, not
the market
value.
5. Depreciation is a non-cash expense. It does not involve any
cash outflow.
Causes of :–
Depreciation
1. By Constant Use.
2. By
Obsolescence
3. By expiry of time.
4. By Accident.
5. By expiry of legal
rights.
6. By Depletion
7. By permanent fall in market
price.
Need, Importance or objects of providing :–
depreciation
1. For ascertaining the truth profit or
loss.
2. For showing the truth ‘true and fair view’ of the financial
position.
3. To ascertain the accurate cost of
production.
162

footer
ACCOUNTING FOR
MANAGERS
4. To provide funds for replacement of
assets.
5. To prevent the distribution of profits out of
capital.
6. For avoiding over payment of Income
tax.
7. Other
objectives.
Factors determining the amount of :–
Depreciation
1. Total Cost of the
Asset.
2. Estimated life of
Asset.
3. Estimated Scrap
Value.
Methods of providing or Allocating :–
Depreciation
1. Straight Line
Method.
2. Written Down Value
Method.
3. Annuity Method.
4. Depreciation Fund
Method.
5. Insurance Policy
Method.
6. Revaluation
Method.
7. Depletion Method.
8. Machine hour rate
Method.
Q. Explain Straight Line Method of Depreciation with the help of
an Example.
Ans. This
Straight
method is also
Linetermed as
:– Original
Cost Method
Method because under this method depreciation is charged at a
fixed
percentage on the original cost of the asset. The amount of
depreciation
remains equal from year to year and as such this method is also known
as
‘Equal Installment Method’, or ‘Fixed Installment Method’. Under this
method,
the amount of depreciation is calculated by deducting the scrap value from
the
original cost of the asset and then by dividing the remaining balance by
the
number of years of its estimated
life. Original Cost of the Asset – Estimated Scrap
Yearly Depreciation Value
———————————————————————
= — Estimated Life of the
Asset.
Merits of Straight Line
Method :–
1. Calculation
Simplicityof :Depreciation under this method is
very –simple and as such the method is widely
popular.
2. Equality of Depreciation Under this method, equal amount
Burden :– is debited to profit & loss account
depreciation of of each year. Hence,
the
burden of depreciation on each year’s net profit is
equal.
3. Assets can be completely written Under this method, the book
off :– of an asset can be reduced to net scrap value or zero value, which
value
is
not possible under some other
methods.
163

footer
4. Knowledge of original cost and upto date under this
depreciation
method, :–
the original cost of the asset is shown in the Balance Sheet
and
the upto-date depreciation is shown as a direct deduction from
it.
Demerits of Straight Line :–
Method
1. When
Difficulty
there are different machines
in
havingdifferent
Computation :–
life-span, the computation of depreciation becomes
complicated
because depreciation on each machine will have to be
calculated
separately
.Unequal pressure in later
2. Repairs charges go on increasing
years
by year:–as the asset becomes older but as theyear
equal depreciation
is
charged under this method each
3. year.
Omission of Interest This method does not take
factor :–
consideration the loss of interest on the amountinto invested in the
asset.
4. Unrealistic to write off the vale of asset to Sometimes, even
zerovalue
the :– of an asset is reduced to zero in the books, it continues afterto be
used
in the business in actual
practice
5. Difficulty in the determination of scrap It is quite difficult
value
assess :–
the true scrap value of the asset after a long period, say 15to
or
20
years from the date of its
installation.
Suitabilit :– This method is suitable for those assets whose useful life can
y
renewals be
.Example :–

Birla Cotton Mills purchased a machinery on 1 May, 1991 for Rs. 90,000. On
stst

1July, 1992 it purchased another machinery for Rs.


40,000.
On 31 March, 1993 it sold off the first machine purchased on 1991 for
st

Rs.
58,000 and on the same date purchased a new machinery for Rs.
1,00,000.
Depreciation is provided at 20% p.a. on the original cost method. Accounts
are
closed each year on 31 December. Show the Machinery Account for three
st

years Dr. Machinery Account


Cr.
Date Particulats J.F. Amount Date Particulars J.F. Amount

1991 1991
May 1 To Bank A/c 90,000 Dec.31 By depreciation A/c 12,000
(for 8 months)
Dec.31 By Balance C/d 78,000

90,000 90,000

164

footer
ACCOUNTING FOR
MANAGERS
1992 1992
Jan1 To Balance B/d 78,000 Dec31 By Depreciation A/c
July1 To Bank A/c 40,000 (i) 18,000
(ii) 4,000 22,000
(for 6 months)
Dec31 By Balance C/d
(i) 60,000
(ii) 36,000 96,000

1,18,000 1,18,000

1993 1993
Jan1 To Balance B/d Mar.31 By Bank A/c 58,000
(i) 60,000 Mar.31 By Dep. A/c 4,500
(ii) 36,000 96,000 (for 3 months)

Mar. To Bank A/c 1,00,000 Dec. 31 By Dep. A/c


31 (ii) 8,000 23,000
Mar. To Profit & Loos (iii) 15,000
31 A/c (Profit On
machine)
Rs. 58,000+
4,500-60,000 2,500 Dec.31 By Bal. C/d
(ii) 28,000
(iii) 85,000 1,13,000

1,98,500 1,98.500

1994
Jan.1 To Bal. B/d 1,13,000
(ii) 28,000
(iii) 85,000

Q. Discuss the Merits And Demerits Of Providing Depreciation


By Diminishing Balance
Method?
Ans. Under
Writtenthis method,
Down as Value
the value of :–
asset on
goes Method
diminishing year after year, the amount of depreciation charged
every
year also goes on
declining.
165

footer
For Example if a machine is purchased for Rs. 10,000 and depreciation is to
charged at 10%bep.a. according to written down value method, the
depreciation
will be charged as
under:-
1 Year on Rs. 10,000 @ 10% =1,000
st

2 Year on Rs. 9,000 (10,000-1,000) @ 10% = 900


nd

3 Year on Rs. 8,100 (9,000-900) @ 10% = 810


rd

and so on.
It will be observed from the above calculations that each year’s depreciation
is
calculated on the book value of the asset at the beginning of that year,
rather
than on the original cost. As the value of asset and also the
depreciation
charged on its goes on reducing year after year, this method is known
as
‘Reducing Installment
Method’.
Merits of Written Down Value :–
Method
1. It isEasy
easyCalculation
to calculate:–the depreciation under
this method, even if some new assets are purchased year after
year.
2. Equal Charge against In this method, the total burden
income&:–Loss account in respect of depreciation
profit on and repairs put
together
remains almost equal year after
3. year.
No induce pressure in later The efficiency of machine is more
years
the :– years than in later years. Hence, theindepreciation in first
earlier
few
years should be more in comparison to the later
years.
4. Balance of asset is never written off to This method ensures
zeroassets
the :– is never reduced to that
zero.
5. Approved method by Income Tax This method of
Authorities depreciation
providing :– is permissible under Income Tax
Regulations.
Demerits of Written Down Value :–
Method
1. Asset can not be completely written Under this method, the
off.an:–asset, even if it becomes obsolete and useless, cannot
of value
be reduced
to
zero and some balance, however small, would continue on asset
account.
2. Omission of Interest This method does not take
Factor :–
consideration into invested in the
the loss of interest on the amount
asset.
3. Difficulty in determining the rate of Under
depreciation
method, the :–
rate of providing depreciation cannot be easily this
decided.
4. Knowledge of original cost & up to date depreciation not
possible :– method, the original cost of various assets is not shown in
Under this
the
Balance
Sheet.
Example :– A company had bought machinery for Rs. 100000 including
there
166

footer
ACCOUNTING FOR
MANAGERS
in a boiler worth Rs 10000 depreciation was charged on reducing
balance
method at the rate of 10% p.a. for first five year and machinery account
was
credited accordingly. During the fifth year, the boiler becomes useless
on
account of damages. The damaged boiler is sold for Rs. 2000 prepares
the
machinery account for five
years.
MACHINERY ACCOUNT
Dr. Cr.
Date Particulars Amount Date Particulars Amount

Year To Bank A/c 90000 Year By Dep.


Ist To Bank A/c 10000 Ist (i) 9000 10000
(Boiler) (ii) 1000
By Bal. C/d
(i) 81000
(ii) 9000 9000
100000 100000
Year To Bal. B/d Year By Dep.
II (i) 81000 II (i) 8100
(ii) 9000 (ii) 900 9000
90000 By Bal. C/d
(i) 72900 81000
(ii) 8100
90000 90000
Year To Bal. B/d Year By Dep.
III (i) 72900 III (i) 7290
(ii) 8100 (ii) 810
81000 By Bal. C/d 8100
(i) 65,610
(ii) 7290
81000 81000
Year To Bal. B/d Year By Dep.
IV (i) 65610 72900 IV (i) 6561 7290
(ii) 7290 (ii) 729
By Bal. C/d
(i) 59049 65610
(ii) 6561
72900 72900
Year To Bal. B/d Year By Bank 2000
V (i) 59049 V By P & L A/c 4561
(ii) 6561 65610 (6561-2000)
By Dep. 5905
By Bal. C/d 53144
65610 65610
Year To bal B/d 53144
VI

167

footer
Q. What do you mean by Final Accounts? What is their
Necessity?
Ans. Financial
Final Statements:– refers to such statements
which
report Accounts
the profitability and the financial position of the business at the end
of
accounting period. The term financial statements include the
following:-
(1) Trading
Account
(2) Profit and Loss
Account.
(3) Balance
Sheet
(1) Trading
Trading account
Account is prepared
:– for calculating the
gross profit or gross loss arising or incurred as a result of the trading activities
a business. In other words, it is prepared to show the result
of
of
manufacturing, buying and selling of
goods.
Need and Importance of Trading :–
Account
(i) It provides information about Gross Profit and Gross
Loss.
(ii) It provides information about the direct
expenses.
(iii) Comparison of closing stock with those of the previous
years.
(iv) It provides safety against possible
losses. Format of a Trading Account: Trading
Account
(for the year ending
————————————)
Dr. Cr.
Particulars` Amount Particulars Amount
Rs. Rs.
To Opening Stock By Sales
To Purchases Loss Sales Return
Less : Purchase Reture OR
OR Return Outward Returns In wards
To Wages By Closing Stock
To Wages & Salaries By Gross Loss (if any)
To Direct Expenses Transferred to P & L A/c
To Carriage or (Balancing Figure)
To Carriage Inwards or
To Carriage on Purchase
To Gas, Fuel and Power
To Freight, Octroi and Cartage
To Manufacturing Expenses
or Productive Expenses.
To Factory Expense, Such as
Factory Lighting, Factor Rent Etc.
To Dock Charges
To Import duty or Custom Duty
To Royalty
To Gross Profit
Transferred to P & L A/c
(Balancing Figure)

168

footer
ACCOUNTING FOR
MANAGERS
(2) Trading
Profit account
& only Lossdisclose:–the gross
profit earned
Accountas a result of buying and selling of goods. However, a
businessman
has to incurr a number of expenses which are not taken into
trading
account. Hence a businessman is more interested in knowing the
net
profit earned or net loss incurred during the
year.
A profit and loss account is an account into which all gains and losses
collected, in order to ascertain the excess of gains over the losses or
are
vice-
versa
.
Need and Importance of Profit & Loss :–
Account
(i) To Ascertain the Net Profit & Net
Loss
(ii) Comparison with previous year’s
profit.
(iii) Control on
Expenses
(iv) Helpful in preparation of the balance
SheetFormat of Profit And Loss Account : Profit And Loss
A/c ( for the year ending __________________)
Particulars` Amount Particulars Amount
Rs. Rs.
To Gross Profit B/d By Gross Prfit B/d
(transferred from trading A/c) (Transferred from trading
A/c)
To Office Expenses :
To Salaries By Rent form tenant
To Salaries & Wages By Discount Received
To Rent, Rate and Taxes By Commission Received
To Printing & Staionery By Any Other Income
To Lighting By Net Loss (if any)
To Telephone Charges Transferred to Capital A/c
To Audit Fees etc.

To Selling & Distribution


Expense:
To Carriage outward or Carriage
on sales
To Advertisement
To Commission
To Bed-Debts
To Export Duty
To Parcking Exp etc.

To Miscellaneous Expenses :
To Discount/Discount Allowed
To Repairs
To Depreciation
To Interest
To Bank Charges etc.
To Net Profit
(Transferred to Capital A/c)

169

footer
(3) After
Balance
ascertaining the net profit or net loss of
the Sheet :– enterprise, the businessman would also like to know the
business
exact
financial position of his business. For this purpose a statement
is
prepared which contains all the assets and liabilities of the
business
enterprise. The statement so prepared is called a Balance
Sheet. Balance
(As on SheetOr As At
--------------)
Particulars` Amount Particulars Amount
Rs. Rs.
Current Liabilities : Current Assets :
Bank Overdraft Cash-in-Hand
Bill Payable Cash at Bank
Sundry Creditors Bills Receivables
Outstanding Expenses Short Term Investments
Unearned Income Sundry Debtors
Closing Stock
Fixed Prepaid Expenses
Liabilities
Long Term:Loans Accrued Income

Reserves: Fixed Assets:


Furniture
Capital: Loose Tools
Add: Net profit Motor Vehicle
Less: Drawings Long-term investments
Less: Income Tax Plant & machinery
Less: Life Insurance Premium Land & Building
Less: Net Loss Patents
Goodwill

Need and Importance of Balance


Sheet
1. The:–main purpose of preparing balance sheet is to ascertain the
true financial position of the business at a particular point of
time.exact information about the exact amount of capital at the end
2. It gives
of the year and the addition or deduction made into it in the current
year in finding out whether the firm is solvent or
3. It helps
not.
4. It helps in preparing the opening entries at the beginning of the next
year.
Q. What is the necessity of doing adjustments? Give some
adjustment
entries with their
explanation.
Ans. InAdjustment
order to ascertain
:– the true profit or loss of the
for a particular
business
s year, it is necessary that all expenses and incomes relating
to
that year are taken into consideration. For example, if we want to ascertain
the
net profit for the year ended on 31 December stand rent for the month
of
170

footer
ACCOUNTING FOR
MANAGERS
December has not yet been paid, it would be proper to include such rent
along
with the other expenses of the year. Similarly, it often happens that
certain
incomes, like interest, dividend, etc. are earned but not received during
year.
the Adjustment for such incomes must be made in the current year itself,
so
that the profit and loss account may disclose the correct amount of net profit
or
loss and the balance sheet may present the true financial position of
the
business
.Simply stated, while preparing final accounts it must be detected whether
there
is a
transaction
(i) Which has been omitted to be recorded in the books,
(ii) Which has been wrongly recorded in the books,
or
or
(iii) Of which only one aspect has been recorded in the
Entriesbooks.
passed for such transactions are called ‘adjustment
entries.’
Need of :–
Adjustments
(1) To ascertain the true Net Profit or loss of the
business.
(2) To ascertain the true financial position of the
(3) To make a record of the transactions omitted from the
business.
books
(4) To rectify the errors committed in the books of
accounts
(5) To make a record of such expenses which have been accrued
but have not been paid.
(6) To make a record of such incomes which have accrued but have
not been
(7) Toreceived.
provide for depreciation and other
provisions. of
Explanation Important :–
Adjustments
(1) The
Closing
amountStock
of goods
:– unsold at the end of the year
is called closing stock. It is valued at Cost Price or Realisable
Value,
whichever is less. The basic principle underlying the valuation of
stock is that anticipated losses should be taken into account, but
closing
all
unrealized gains should be
ignored.
Treatment in Final
Accounts :–
(i) If the closing stock appears outside the Trial Balance, it will be
shownat two places, i.e., on the Credit side of the Trading A/c and on
Assets side of the Balance
the
(ii) If Sheet.
the closing stock appears inside the Trial Balance, it will be
shownonly on the Assets side of the Balance
(2) Sheet. Expenses Or Expenses Due but not
Outstanding These are
Paid :– which have been incurred during the year but have been
expenses the
on the date of preparation of final
unpaid
accounts.
Treatment in Final
Accounts :–
(i) If outstanding expenses have been mentioned inside the
Trial
171

footer
Balance, they will be shown on the liabilities side
only.
(ii) If outstanding expenses have been mentioned outside the
Trial Balance, then on the one hand, it will be added to the
concerned
expenses on the debit side of Trading or Profit and Loss Account
and
on the other hand, will also be shown on the liabilities side of
the
Balance
Sheet.
(3) Prepaid expenses Or Unexpired Expenses Or Expenses Paid
Advance in :– These are the expenses which have been paid in advance for
next year during the the current year
itself. Treatment in Final
Accounts :–
(i) If Prepaid expenses have been mentioned inside the Trial
Balance,
they will be shown on the Assets side
(ii) Ifonly.
Prepaid expenses have been mentioned outside the Trial
Balance,
then on the one hand, it will be deducted from the
concerned
expenses on the debit side of Trading or Profit and Loss Account
and
on the other hand, will also be shown on the Assets side of
the
Balance
Sheet.
(4) Depreciation
Depreciation is the
: loss or fall in the value of fixed
assets –due to their constant use and expiry of
time.
Treatment in Final Depreciation on the one hand, will
Accounts
shown on :– the debit side of the Profit and be
Loss Account and on the
other
hand, will also be deducted from the value of the concerned asset on
the
Asset side of the Balance
Sheet.
(5) Accrued Income or Income It is quite common that
Receivable
items of income:– such as interest, commission etc are certain
earned during
the
current year but have not been actually received by the end of the
current
year. Such incomes are known as ‘Accrued Incomes’ or ‘Earned
Incomes’
Treatment in Final
Accounts :– incomes have been mentioned inside the Trial
(i) If accrued
Balance,
they will be shown on the Assets side
(ii) Ifonly.
Accrued incomes have been mentioned outside the Trial
Balance,
then on the one hand, It will be shown on the credit side of the Profit
&
Loss Account and on the other hand, will be shown on the assets
side
of the Balance
Sheet.
(6) Unearned Income Or Income Received in It may
Advancethat
happen :– a certain income is received in the current year but the also
whole
amount of it does not belong to the current year. Such portion of
this
income which belongs to the next year is known as Unearned Income
or
Income received but not
earned.
172

footer
ACCOUNTING FOR
MANAGERS
Treatment in Final
(i) Accounts
If Unearned:– incomes have been mentioned inside the Trial
they will be shown on the Liabilities side
Balance,
(ii) Ifonly.
Accrued incomes have been mentioned outside the Trial
then on the one hand, It will be deducted from the concerned
Balance,
on the Credit side of the Profit & Loss Account and on the other
income
will be shown on the Liabilities side of the Balance
hand,
(7) Usually Sheet.
Interest
in order to ascertain
on the true efficiency
of Capital
the :–
business, interest at a normal rate is charged on the capital
by the proprietor in the
invested
business.
Treatment in Final Interest on capital is an expense for
business and
Accounts :– hence it is shown on the the debit side of Profit & Loss
At the same time, it is a gain to the proprietor and hence is added to
Account.
capital
his
.Interest
(8) Occasionally, the proprietor
on draws cash or
goods for his personal
Drawings :– use. Such withdrawals are terms as Drawings. If the
pays interest on capital, it is fully justified that it should also
firm
interest
charge on
drawings.
Treatment in Final Interest on drawings is a gain to
business
Accountsand :– hence it is shown on the credit the side of Profit & Loss
At the
Account. same time, it is an expense from the proprietor’s view and hence
be deducted from the
will
(9) capital.
Interest on
Loan :–
(i) Generally, item of Loan appears on the credit side of the
Trial Balance. It means that the amount has been borrowed from
person or the bank etc. Loan is a liability of the firm and the
some
on such loan will be an expense. It up-to-date interest has not
interest
paid
been on the Loan, the unpaid interest will have to be calculated
and
will be treated just like outstanding
expenses.
Treatment in Final Accounts :– When Loan appears on the credit
of the Trial Balance, interest on it will be an expense and hence will
side
be
recorded on the debit side of Profit & Loss Account.
amount of such interest will also be added to Loan Account on
Outstanding
Liabilities
the side of the Balance
(ii) OnSheet.
the contrary, if the item of loan appears on the debit side of
Trial Balance, it will mean that the amount has been lent to outsider. It
be an asset in this case and interest on such loan will be an
will
for
income the
firm.
Treatment in Final When Loan appears on the
side of the:–Trial Balance, interest on it will
Accounts be an income and
Debit
hence
will be recorded on the credit side of Profit & Loss Account and
also be added to Loan Account on the assets side of the
will
Sheet.
Balance

173

footer
(10) Persons
Bad Debts to :–
whom goods have been sold on credit are
knownas Debtors. Sometimes due to the dishonesty, death or insolvency of
adebtor, full amount is not received from him. When it becomes
certain
that a particular amount will not be recovered it is known a s ‘ B a d
-Debts’
.Treatment in Final
Accounts :–
(i) If Bad-debts are given in the adjustments or outside the
Trial Balance, they will be shown on the debit side of the Profit &
Loss
Account and will also be deducted from the Debtors on the assets
side
of the Balance
(ii) If Sheet.
Bad-Debts are given inside the trial balance, it will be shown on
the debit side of the Profit & Loss
Account.
(11) Provisions for Bad and Doubtful Even after deducting
amount
Debts :–of actual bad-debts from the debtors, the list of debtors
the at the
end
of the year include some debts which are either bad or doubtful.
A
provision is created to cover any possible loss on account of bad-
debts
likely to occur in future. Such a provision is created at a fixed
percentage
on debtors every year and is called ‘provisions for bad and doubtful
debts’.
Treatment in Final Accounts :– The amount of provision for doubtful
debts
on the one hand, is shown on the debit side of the Profit and
Loss
Account and on the other hand, is deducted from Sundry debtors on
the
assets side of the Balance
Sheet.
(12) Provisions for Discount on It is a normal practice in
Debtors :–
business to allow cash discount to those debtors the from whom the
is received promptly or with a fixed period. Discount thus allowed will
payment
be
an expense of the business. It should be noted that discount will
be
allowed only to those debtors who will make prompt
payment.
Treatment in Final Such provision is shown on the
Accounts
side of the:– profit & loss account and is debit
also deducted from
Sundry
Debtors on the Assets side of the Balance
(13) Sheet.
Provisions for Discount on Such provision is shown on the
Creditors
credit side :–
of the Profit & Loss account and is also deducted from
the
Sundry Creditors on the Liabilities side of the Balance
Sheet.
(14) Abnormal
Sometimes Loss losses
:– occur due to some
abnormal
circumstances such as accident, fire, flood, earthquakes etc. Such
are called abnormal losses. These may be divided into two
losses
categories:
(i) Loss of Goods :– It will be that on the one hand, the loss of goods
will deducted from the purchase on the Debit side of Trading Account
and
it will also be shown on the debit side of Profit & Loss
Account
(ii) Loss of Fixed Assets :– If some fixed assets of the firm is destroyed
by
174

footer
ACCOUNTING FOR
MANAGERS
some accident, then the loss will be shown on the debit side of
P&L
A/c and also deducted from the value of Asset on the assets side
of
the Balance
Sheet.incertain
(15) Occasionally,
Charity the amount
Form of of
goods
is Goods :– as charity. On the one hand, the amount will be deducted
given away
from
purchase and on the other hand it will also be shown on the debit side
of
P&L A/c.
(16) Goods Distributed as Free Sometimes the goods which
Samples deals
business :– in are distributed as free samples forthe
the purpose
advertising these goods. On the one hand, the amount will be
of
deducted
from purchase and on the other hand it will also be shown on the
debit
side of P&L
(17) IfA/c.
Drawings
the proprietor
in Goods
of the
:– business has taken
some goods for his personal use from the business, it is known as Drawings
in
Goods. It will be deducted from purchase in the Trading Account and
will
also be deducted from the Capital on the liabilities side of the
Balance
Sheet as
(18) Drawings.
Deferred Revenue There are certain expenditures
Expenditure
are revenue in:–nature but the benefit of whichwhich
is likely to be derived over
anumber of years. Such Expenditures are termed as ‘Deferred
Expenditure’. As such, the whole of such expenditure is not debited to
Revenue
the
Profit and Loss Account of the current year but spread over the years
for
which the benefit is likely to last. Thus, only a part of such expenditure
is
taken to Profit & Loss Account every year and the unwritten off portion
allowed to stand on the assets side of the Balance
is
Sheet.
(19) Manager’s Commission on Net Sometimes, in addition to
Profit :–salary, the manager is entitled to a commission
regular his on net
profit.
Treatment in Final On the one hand, it will be recorded
Accounts
the debit :–
side of P& L A/c and on theonother hand, shown on the
liabilities
side as an outstanding
expense.
Methods of Calculating the
Commission
(i) On Profits:–before charging such commission: The formula
is: Rat
Manager’s Commission = Net Profit x e
———— 100
(ii) On Profits after charging such commission: The formula
is: Rat
Manager’s Commission = Net Profit ex
———————— 100 + Rate

175

footer
ACCOUNTING FOR
MANAGERS
MBA 1st Semester
(DDE)

UNIT – II

Q. What is Ratio Analysis? Explain its Objectives and Limitations.


Also give its
classification.
Ans. Ratio
Absolute:–figures expressed in monetary terms in
financial
statements by themselves are meaningless. These figures often do not
convey
much meaning unless expressed in relation to other figures. Thus, we can
say
that the relationship between two figures, expressed in arithmetical terms
is
called a
‘ratio.’
According to R.N. Anthony
A ratio is simply one number expressed in terms of another. It found by
dividing
one number into the
other.
Ratio Analysis discloses the position of business, so it is a very important tool
of
financial analysis. But ratio analysis suffers from a no. of limitations.
These
limitations should be kept in mind while making use of the Ratio
Analysis.
Objectives of Ratio :–
Analysis
(1) Helpful in Analysis of Financial Ratio analysis is
Statements :– device for analyzing the financial statement. It helps
extremely useful an
the
bankers, creditors, investors, shareholder etc. in acquiring
enough
knowledge about the profitability and financial health of the
business.
(2) Simplification of Accounting Accounting ratio simplifies
Data :–
summarizes a long array of accounting data andand makes
them
understandable. It discloses the relationship between two such
figures
which have a cause and effect relationship with each
other.
(3) Helpful in Comparative With the help of ratio
Study :–
comparison analysis can be made
of profitability and financial soundness
between
one firm and another in the same industry. Similarly, comparison
of
current year figures can also be made with those of previous years with
the
help of ratio
analysis.

176

footer
ACCOUNTING FOR
MANAGERS
(4) Helpful in Locating the Weak Spots of the Current
Business
ratios are :–compared with those of the previous years and if some year’s
weak
spots are thus located, remedial measures are taken to correct
them.
(5) Accounting
Helpful ratios are very in helpful
in Forecasting
forecasting and:– preparing the plans for the
future.
(6) Estimate about the Trend of the If accounting ratios
Business for
prepared :– a number of years, they will reveal the trend of arecosts,
sales,
profits and other important
facts.
(7) Fixation of Ideal Ratio helps us in establishing
Standards
standards of:– the different items of the business.
ideal By comparing the
actual
ratios calculated at the end of the year with the ideal ratios, the
efficiency
of the business can be easily
measured.
(8) Ratio
Effective
Analysis discloses the liquidity, solvency
and Control :– of the business enterprise. Such information
profitability
enables
management to assess the changes that have taken place over a period
of
time in the financial activities of the business. It helps them in
discharging
their managerial functions, e.g. planning, organizing,
directing,
communicating and the controlling more
effectively.
(9) Study of Financial Soundness :– Ratio analysis discloses the position
of
business with different view-points. It discloses the position of
business
with the liquidity point of view, solvency point of view, profitability point
of
view etc. With the help of such a study we can draw conclusions
regarding
the financial health of the business
enterprise.
Limitations of Ratio :–
Analysis
1. False accounting date gives false Accounting ratios
ratios :– on the basis of data given in profit & Loss account
calculated are and
balance-
sheet. There are certain limitations of financial statements, and hence
the
ratios calculated on the basis of such, financial statements will also
have
the same
limitation.
2. Comparison not possible if different firms adopt different
accounting
policies : There may be different accounting policies adopted by
–firms with regard
different
to providing depreciation etc. For example, one firm
may

177

footer
adopt the policy of charging dep. On Straight Line Method, while
other
may charge on written-down-value method. Such difference makes
the
accounting ratios
incomparable.
3. Ratio Analysis becomes Less Effective Due to Price Level
Changes :– over the years goes on changing, therefore, the ratios of
Price level
various
years cannot be
compared.
4. Ratios may be misleading in the absence of absolute For e.g. X
data :–
Co. produces 10 Lakh meters of cloth in 1992 and 15 Lakh meters in
1993,
the progress is 50%. Y Co. raises production from 10 thousand meters
in
1992 to 20 thousand meters in 1993, the progress is 100%. Comparison
of
these two firms made on the basis of ratio will disclose that the second
firm
is more active that the first firm. Such conclusion is quite
misleading
because of the difference in size of the two firms, it is therefore essential
to
study the ratios along-with the absolute data on which they are
base.
5. Limited Use of a Single The analyst should not merely rely on
Ratio
single :– a
ratio. He should study several connected ratios before reaching
aconclusion.

6. Circumstances
Lack of Proper differ
Standard
from firm
:– to firm
hence no single standard ratio can be fixed for all the firms against which
the
actual ratio may be
compared.
7. Ratios alone are not adequate for Proper Ratios
Conclusions
from analysis:–of statements are not sure indicators of good or derived
bad
financial position and profitability of a firm. They merely indicate
the
probability of favorable or unfavorable position. The analyst has to
carry
out further investigations and exercise his judgment in arriving at
acorrect
diagnosis.
8. Effect of Personal ability and bias of the Another
Analyst :– in mind is that different persons draw different meaning
point to keep important
of
different terms. One analyst persons draw different meaning of
different
terms. One analyst may calculate ratios on the basis of profit after
interest
and tax, while other may consider profit after interest but before
tax
Classification of :– Ratios may be classified into the four
Ratios categories.with the help of following
Classification of ratios can be explained
diagram:
178

footer
ACCOUNTING FOR
MANAGERS
Classification of
Ratios

Liquidity Ratios Leverage Or Activity Or


Profitability
Or Short-term Capital Structure Turnover
Solvency
Ratios Ratios Ratios
Ratios
Current Ratio Stock Turnover
Ratio
Liquid Ratio Debtors Turnover
Ratio Average Collection
Period
Debt Equity Ratio Creditors Turnover
Ratio
Debt to Total Funds Average Payment
Period
Ratio Fixed Assets
Turnover
Proprietary Ratio
Ratio
Fixed Assets to Working
Capital
Proprietor’s fund Ratio Turnover
Ratio
Capital Gearing
Ratio
Interest Coverage
Ratio

Profitability Ratios Profitability Ratios


based on Sales on
Investment
Gross Profit Ratio Return on Capital
Employed
Net Profit Ratio Return on Shareholder’s
Fund
Operating Ratio (i) Return on
Total
Expenses Ratios Shareholder’s
Funds (ii) Return on
Equity’s
Shareholder’s
funds
(iii) Earning Per
Share
(iv) Dividend per
Share
(v) Price Earning
Ratio
Q. Explain the Important Ratios calculated for Evaluating the
Short-
Term Solvency Position of a Company.
OR
Q. Explain the Liquidity Ratios in
detail.
Ans. “Liquidity”
Liquidity refers to the:–ability of the firm to meet
its Ratios
current liabilities. The liquidity ratios, therefore, are also called ‘Short-
term
Solvency Ratios.’ These ratios are used to assess the short-term
financial
position of the
concern.
179

footer
Liquidity ratios include two
ratios
This:– ratio explains the relationship between current
1. Current
assets
andRatio:–
current liabilities of a business. The formula for calculating the ratio
is:
Current
Current Ratio Assets =
—————————————— Current
Current Current assets includeLiabilities
those assets which can be
Assetscash
into :– within a year’s
converted
time.
CONSTITUENTS OF CURRENT ASSETS
1. Cash-in-hand and Bank
balances
2. Bills
Receivables
3. Sundry Debtors (less provision for bad
debts)
4. Short-term Loans and
Advances
5. Inventories of Stock,
as :
(a) Raw
materials,
(b) Work-in
process
(c) Stores and
spares
(d) Finished goods

7. Prepaid
Expenses
8. Accrued
Incomes
Current Liabilities :– All liabilities which are payable within one year
are
known as current
liabilities.
CONSTITUENTS OF CURRENT
LIABILITIES
1. Bills
Payables
2. Sundry Creditors or Accounts
Payable
3. Accrued or Outstanding
Expenses
4. Short-term Loans, Advances and
Deposits.
5. Dividends
Payables.
6 . Bank
Overdraft
7. Provision for Taxation, if it does not amount
to appropriation of
profits

180

footer
ACCOUNTING FOR
MANAGERS
Ideal According to accounting principle, a current ratio of 2:1
Ratio :– to be an isideal ratio. It means that current assets of a
supposed
should, atleast, be twice of its current liabilities. The reason of assuming 2: 1
business
as
the ideal ratio is that the current assets include such assets as stock,
debtors
etc., from which full amount cannot be realized in case of need. Hence,
even
half the amount is realized from the current assets on time, the firm can
still
meet its current liabilities in
full.
Significance : This ratio is used to assess the firm’s ability to meet its

term short-
liabilities on time. According to accounting principle, a current ratio of
2:1
is assumed to be an ideal ratio. If the current ratio is less than 2:1, it
indicates
lack of liquidity and shortage of working capital. But a much higher ratio,
though it is beneficial to the short-term creditors, is not necessarily good for
even
the
company. A much higher ratio than 2:1 may indicate the poor
investment
policies of the management. A much higher ratio may be considered to
be
adverse from the view point of management on account of the
following
reasons
:2. Liquid
Liquid ratio
Ratio
explains
:– the relationship between
liquid assets and current liabilities of a business. The formula for calculating
the
ratio
is :– Liquid
Liquid RatioAssets =
————————————- Current
Liabilities
Liquid Liquid assets include those assets which will yield cash
Assets :–All current assets
shortly. very except stock and prepaid expenses are included
in
liquid
assets.
CONSTITUENTS OF LIQUID ASSETS
1. Cash-in-hand and Bank
balances
2. Bills
Receivables
3. Sundry Debtors (less provision for bad
debts)
4. Short-term Loans and
Advances
5. Temporary Investment of Surplus
Funds
6. Accrued
Incomes

OR Liquid Assets= Current Assets- Stock – Prepaid


Expenses
Ideal According to accounting principle, a liquid ratio of 1:1
supposed
Ratio :– to be an ideal
is ratio. It means that liquid assets of a business
should,
atleast, be equal to its current liabilities. The higher the ratio, the better it
is,
because the firm will able to pay its current liabilities more
easily.
181

footer
Significance : An ideal Liquid ratio is said to be 1:1. If it is more, it

considered is The idea is that for every rupee of current
to be better.
liabilities,
there should atleast be one rupee of liquid assets. This ratio is a better test
of
short-term financial position of the business other than the current ratio, as
it
considers only those assets which can be easily and readily converted
into
cash. Liquid ratio thus is a more rigorous test of liquidity than the current
ratio
and, when used together with current ratio, it gives a better picture of the
short-
term financial position of the
business.
Q. Explain the Important Ratios Calculated for Evaluating the Long
- Term Solvency Position of a Company.
OR
Q. Explain the Capital Structure Ratios in
detail
Ans. These
Capital
ratios areStructure
calculated to assess
:–
the Ratios
ability of the firm to meet its long term liabilities when they become due.
Long
term creditors including debenture holder and primarily interested to
know
whether the co. has ability to pay regular interest due to them and to repay
the
principal amount when it become due. These ratios includes the
following
ratios:
- These ratios include the
following:
1. Debt Equity Ratio :–
Debt Long term
Debt Equity Loans
Ratio= ————— OR
—————————————Equity Shareholder’s
funds
Debt :– These refer to long-term liabilities which mature after one year.
These
include Mortgage Loan, Debenture, Bank Loan, Loan from
financial
institutions, Public Deposits
etc.
Shareholder’s Equity Share Capital, Preference Share
Funds :– premium, General Reserve,
Securities capital,
Capital Reserve, other reserves
and
credit balance of profit & loss
a/c.
However, accumulated losses and fictitious assets remaining to the written
off
like preliminary expenses, underwriting commission, share issue expense
etc,
should be deducted.
Significance : This ratio is calculated to assess the liability of the firm to
– long-term liabilities.
its meetGenerally, debt equity ratio of 2:1 is considered safe.
If
the debt equity ratio is more that that, it shows a rather risky financial
position
from the long term point of view, as it indicates that more and more
funds
invested business are provided by long-term lenders. A high debt equity ratio
is
a danger-signal for long-term
lenders.
182

footer
ACCOUNTING FOR
MANAGERS
2. Debt to Total Funds Ratio :–
Debt Long term loans
Debt to total funds ratio= ————— OR
————————————————————
Debt+ Equity long term loans+ shareholder’s Funds

Significance : Generally, debt to total fund ratio is (.67:1) is



satisfactory. In other considered
words, the proportion of long term loans should not
more
than 67% of total funds. A high ratio than this is generally treated an
indicator
of risky financial position from the long-term point of view, because it
means
that the firm depends too much upon outside loans for its
existence.
3. Proprietary
Ratios :– Equity
Proprietary Ratio =
—————————— Equity + Debt

Significance : This ratio should be 33% or more than that. In other words,
–proportion of shareholders
the funds to total funds be 33% or more. A
higher
proprietary ratio is generally treated an indicator of sound financial
position
from long-term point of
view.
4. Fixed Assets to Proprietor’s
Ratio :– Fixed
Fixed Assets to Proprietor’s Assets Ratio=
——————————————— Proprietor’s funds (net
worth)
Significance : The ratio indicates the extent to which proprietor’s funds
–sunk into fixed assets.
areNormally, the purchase of fixed assets should
be
financed by proprietor’s funds. If this ratio is less than 100%, it would
mean
that proprietor’s funds are more than fixed assets and a part of working
capital
is provided by the
proprietors.
5. Capital Gearing
Ratio :–
Equity Share Capital+ Reserves + P&L (Cr.) Balance
Capital Gearing Ratio=
———————————————————————————Fixed Cost bearing capital

Fixed Cost bearing capital = Preference share capital+ Debenture+


Long
term
loans
Significance :– A high gearing will be beneficial to equity shareholders when
the
rate of interest/dividend payable on fixed cost bearing capital is lower than
the
rate of return on investment in
business.
183

footer
6. Interest Coverage
Ratio :– Net Profit before interest &
Interest Coverage tax Ratio =
—————————————————— Fixed Interest
Charges
Significance : This ratio indicates how many times the interest charges

covered areavailable to pay interest charges. A long term lenders
by the profits
in
finding out whether the business will earn sufficient profits to pay the
interest
charges regularly. The higher ratio more secure the lender is in respect
of
payment of interest regularly. An interest coverage ratio of 6 to 7 times
is
considered
appropriate.
Q. Explain the Activity Ratios Or Turnover Ratios in
detail.
Ans. These
Activityratios are calculated
:– on the basis of ‘cost of
sales’
or Ratiostherefore, these ratios are also called as ‘Turnover Ratios’.
‘sales’,
Turnover
indicates the speed or number of items the capital employed has been
in the process of doing business. In other words, these ratios indicated
rotated
how
efficiently the capital is being used to obtain sales; how efficiently the
fixed
assets are being used to obtain sales; and how efficiently the working
capital
and stock is being used to obtain sales. Higher turnover ratios indicate
the
better use of capital or resources and in turn lead to higher
profitability.
Turnover ratios include the
following:
1) ThisInventory
ratio indicates whether
Turnoverinventory
has been
Ratio efficiently
:– used or not. This ratio indicates the relationship
between
the cost of goods sold during the year and average stock kept during
that
year. The formula for calculating the ratio is
: Cost of Goods Sold
Inventory Turnover Ratio =
————————————— Average
Stock
Cost of goods sold can be calculated by two
ways :–
Cost of Goods Sold = Sales – Gross
Profit OR
Cost of Goods Sold = Opening Stock + Purchases + Carriage +
Wages
+ Other Direct Expenses – Closing
Stock Opening Stock + Closing
Average Stock Stock =
—————————————————— 2
Significance : This ratio shows the speed with which the stock is rotated

sales or the number into
of times the stock is turned into sales during the year.
The
higher the ratio, the better it is, since it indicates that stock is selling quickly.
In
184

footer
ACCOUNTING FOR
MANAGERS
a business where stock turnover ratio is high, goods can be sold at a low
margin
of profit and even then the profitability may be quite
high.
(2) Inventory
This ratio indicates the
Holding
time within which
the Period
stock :–is converted into sales. This ratio is computed by the
following
formula
: 12months/ 52 weeks/ 365 days
Inventory Holding Period =
—————————————————— Stock Turnover
Ratio
Inventory holding period can be calculated in days or months or
weeks.
(3) This
Debtors
ratio indicates
Turnover
the relationship
between
Ratio
credit :–
sales and average debtors during the year. The formula
for
calculating the ratio
is:
Net Credit
Debtors Turnover Sales
Ratio =
———————————————————— Average Debtors + Average
B/R
Net Credit Sales = Total Sales – Cash
Sales
Opening Debtors + Closing
Average Debtors
Debtors =
———————————————————————— 2

Opening B/R + Closing B/R


Average B/R =
—————————————————
2
Bills receivable are added in debtors for the purpose of calculation of this
ratio.
While calculating this ratio, provision for bad and doubtful debts is
not
deducted from total debtors, so that it may not give a false impression
that
debtors are collected quickly. Debtors turnover ratio can be calculated on
the
basis of total sales instead of credit
sales.
Significance : This ratio indicates the speed with which the amount

collected from debtors. is The higher the ratio, the better it is, since it
indicates
that amount from debtor is being collected more quickly. The more quickly
the
debtors pay, the less the risk from bad debts, and so the lower the expenses
of
collection and increase in the liquidity of the
firm.
(4) This
Average
ratio indicates Collection
the time within
which
the Periodis :–
amount collected from debtors and bills receivable. This ratio can
computed by the following three
be
formulas:

185

footer
First
Formula :–
Average Debtors + Average
Average Collection B/R Period =
—————————————————— Credit Sales per
day
Net Credit Sales of the
Credit Sales Year
per Day =
———————————————— 365

Second Formula :–
Average Debtors x
Average Collection 365
Period =
———————————————— Net Credit
Sales

Third Formula :–
12 months/ 365 days/ 52 weeks
Average Collection Period =
————————————————— Debtor Turnover
Ratio
Significance : This ratio shows the time in which the customers are paying
–credit sales. For example,
for in a business average collection period is 30 days.
It
means that, on an average, if sale is made today, the cash will be
collected
actually after 30 days, i.e., 30 days credit sales are locked up in
debtors.
(5) This
Creditors
ratio indicates
Turnover
the relationship
betweenRatio
credit :–
purchases and average creditors during the year. The formula
for
calculating the
ratio:
Net Credit
Creditors Turnover Purchases
Ratio =
—————————————————— Average Creditors + Average
B/P
Net Credit Purchase = Total Purchases – Cash
Purchase
Opening Creditors + Closing
Average Creditors
Creditors =
——————————————————————— 2

Opening B/P + Closing B/P


Average B/P =
———————————————— 2

186

footer
ACCOUNTING FOR
MANAGERS
This ratio can be calculated on the basis of total purchases instead of
credit
purchases
.Significance : This ratio indicates the speed with which the amount is

paid being
to creditors. The higher the ratio, the better it is, since it will indicate
that
the creditors are being paid more quickly which increases the credit
worthiness
of the
firm.
(6) This
Average
ratio indicates the
Payment
time which
is Period
normally:– taken by the firm to make payment to its creditors. This ratio
can
be calculated by the following three
formulas:
First
Formula :–
Average Creditors + Average
Average Payment B/P Period =
—————————————————— Credit Purchase per
day
Second Formula :–
Average Creditors x
Average Payment 365Period =
——————————————— Net Credit
Purchases
Third Formula :–
12months/ 52 weeks/ 365 days
Average Payment Period =
—————————————————— Creditors Turnover
Significance : This ratio shows the time Ratio
in which the creditors are paid

credit forlower the ratio, the better it is, because a
purchases. The
shorter
payment period implies that the creditors are being paid
rapidly.
(7) Working Capital Turnover This ratio indicates the
Ratio :– cost of goods sold and working capital.
between relationship
The formula
for
calculating the ratio
is:
Cost of Goods Sold
Working Capital Turnover Ratio =
———————————— Working Capital

Working Capital = Current Assets – Current


Liabilities
Significance : This ratio indicates how efficiently working capital has

utilised been This ratio is of particular importance in
in making sales.
manufacturing concerns where current assets play a major role in
non-
generating
sales. This ratio shows the number of times on which working capital has
been
187

footer
rotated in producing sales. A high working capital turnover ratio
shows
efficient use of working capital and quick turnover of current assets like
stock
and debtors.
Q. Explain the Important Ratios Calculated for evaluating
the Profitability of a Company.
OR
Q. Explain the Profitability Ratios in
detail
Ans. The
Profitability
main object of every :– business is to earn
profits.
A Ratiosmust be able to earn adequate profits in relation to the risk
business
and
capital invested in it. The efficiency and the success of a business can
measured with the help of profitability ratios. Profitability Ratios can
be
be
determined on the basis of either sales or investment into
business.
(A) Profitability Ratios Based on :– These ratios include the
Sales
(1) This
Gross
ratio showsProfit
the relationship between following
gross Ratio :– sales. The formula for computing this ratio
profit and
is:
Gross
Profit
Gross profit Ratio= —————————x
100 Net
Gross Profit = Sales – Cost ofSales
Goods
Sold
Net Sales = Sales – Sales
Return.:– This ratio measures the margin of profit available on sales.
Significance
The
higher the gross profit ratio, the better it is. No ideal standard is fixed for
ratio,
this but the gross profit ratio should be adequate enough not only to cover
the
operating expenses but also to provide for depreciation, interest on
loans,
dividends and creation of
reserves.
(2) This
Net ratio shows
Profit
the relationship between net
profit and
Ratiosales.
:– It may be calculated by two
(i methods:
Net Profit
) Ratio :– Net
Net Profit Ratio= ————————Profit
x100 Net
Sales
Net Profit= Gross Profit- All Indirect Expenses + All indirect
Incomes

(ii Operating Net Profit


) Ratio :–
Operating Net
Profit
Operating Net Profit Ratio = —————————————
x100 Net
Sales
188

footer
ACCOUNTING FOR
MANAGERS
Operating Net Profit= Gross Profit- Operating
Expenses
Operating Expenses= Office and Administration Expenses, Selling
and expenses, Bad debts, Discount, Interest on short-term
distribution
debts.
Significance : This ratio measures the rate of net profit earned on sales.

helps in determiningItthe overall efficiency of the business operations.
An
increase in the ratio over the previous year shows improvement in the
overall
efficiency and profitability of the
business.
(3) This
Operating
ratio measures the proportion of an
enterprise’s
Ratio
cost of:–sales and operating expenses in comparison to its
sales: Cost of Goods Sold + Operating
Expenses
Operating Ratio : ————————————————————— X
100 Net
Sales

Cost of goods sold can be calculated by two


ways :–
Cost of Goods Sold = Sales – Gross
Profit OR
Cost of Goods Sold = Opening Stock + Purchases + Carriage + Wages
Other Direct Expenses – Closing
+
Stock
Operating = Office and Administration Expenses, Selling
Expenses Expenses, Badand
distribution debts, Discount, Interest on short-term
debts :
Significance Operating ratio is a measurement of the efficiency

profitability and enterprise. The ratio indicates the extent of
of the business
sales
that is absorbed by the cost of goods sold and operating expenses. Lower
the
operating ratio, the better it is, because it will leave higher margin of profit
on
sales
.
(4) These
Expenses
ratios indicate the relationship
betweenRatios
expenses:– and sales. The ratio may be calculated
as: Material
(i Consumed
Material Consumed Ratio = ————————————— X
) 100 Net
Sales
Direct Labour
(ii Cost
Direct Labour Cost Ratio = —————————————X
) 100 Net
Sales
Factory
(iii Expenses
Factory Expenses Ratio = ———————————— X
) 100 Net
Sales
189

footer
(B) Profitability Ratios Based on Investment in the Thes
Business
ratios :– the true earning capacity of the resources employed in
reflect e
the
enterprise Sometimes the profitability ratios based on sales are
high
whereas profitability ratios based on investment are low. These may
be
classified into two
categories:
(1) Return on Capital
Employed
(2) Return on Shareholder’s
Funds
(1) This
Returnratio reflects
on the
Capital
overall
profitability
Employed
of :– This ratio is also known as ‘Rate of Return’ or ‘Yield
the business.
on
Capital’. The ratio is computed as
under:
Profit before Interest, tax and dividends
Return on Capital Employed = —————————————————— X
100 Capital
Employed
Capital Employed :– This can be computed by any of the following
methods: two
Capital Employed = Debt + Equity – Non Operating
Assets OR
Capital Employed = Fixed Assets + Current Assets – Current
(2) Liabilities
Return on Shareholder’s Return on shareholders
Funds :– only the profitability of the funds
measures funds
invested by
shareholders.
There are several measures to calculate the return on
shareholder’s
funds:
(i Return on Total Shareholder’s The ratio is computed as
) Funds :– under:
Net profit After Interest and Tax
Return on Total Shareholder’s Funds = ———————————————— X
100 Total Shareholder’s
Funds

Total Shareholder’s Funds = Equity Share Capital + Preference


Share
Capital + All Reserves + P&L A/c Balance – Fictitious
assets
Significance : This ratio reveals how profitably the proprietor’s funds

been utilized by the have
firm. A comparison of this ratio with that of similar
firms
will throw light on the relative profitability and strength of the
firm.
(ii Return on Equity Shareholder’s This ratio is computed
)
under:Funds :– as
Net profit After Interest, Tax and Preference Dividend
Return on Equity Shareholder’s Funds = —————————————————— X
100
Equity Shareholder’s
Funds

190

footer
ACCOUNTING FOR
MANAGERS
Equity Shareholder’s Funds = Equity Share Capital + All Reserves +
P&L
A/c Balance – Fictitious
Assets
Significance : This ratio measures how efficiently the equity

funds are being shareholder’s
used in the
business.
(iii) This
Earning
ratio measures
Per the profit
Shareavailable
to (E.P.S.)
the equity :–
shareholders on a per share basis. This ratio is computed as
under:
Net Profit – Dividend on Preference
Earning Shares
Per Share =
——————————————————————— Number of Equity
Significance : This ratio is helpful in the Shares
determination of the market price

the equity shareof of the
company.
(iv) Profit
Dividend
remaining after
Perpayment of tax
and preference
Share :– dividend are available to equity shareholders. But all of
these
are not distributed among them as dividend. Out of these profits, a
portion
is retained in the business and remaining is distributed among
equity
shareholders as
dividend. Dividend Paid to Equity
Dividend Per Shareholders Share =
—————————————————————
Number of Equity
Shares
(v) Dividend Payout Ratio Or D.P. :– This ratio is computed as
under:
D.P.S
D. P. = —————— X 100
E.P.S
(vi) Earning and Dividend Yield :–
EPS
Earnings Yield = ——————————————— X
100 Market Value Per
Share

DPS
Dividend Yield = ———————————————— X
100 Market Value Per
Share

(vii) Price Earnings(P.E)


Ratio :–
Market price of the
P.E. shareRatio =
—————————————————
EPS

191

footer
Q. What is Fund Flow Statement? How is it
prepared?
Ans. Meaning of Fund Flow :– The balance sheet of a firm
Statement
the position of assets, liabilities and capital at the enddiscloses
of a particular year.
But
it does not disclose the causes of changes in these items between the end
of
previous year and the end of current year. Therefore, an additional
statement
called ‘Fund Flow Statement’ is prepared to show the changes in
assets,
liabilities and capital between the dates of two balance
sheets.
Meaning of Funds :– In a limited sense, the term ‘fund’ means ‘cash’. But
is not the correct meaningthis of the term ‘fund’ because there are
many
transactions in the business which do not result in inflow or outflow of cash
but
certainly result in the inflow or outflow of funds. As such, the term ‘fund’
stands
for ‘Net Working
Capital”.
Meaning of Flow :– The term ‘flow’ means change or movement. Therefore,
term ‘Flow of Funds’ means the increase or decrease in working capital. If
atransaction results in the increase of working capital, it is said to be a source
of
funds and if the transaction results in the decrease of working capital, it is
said
to be an application of funds. If the transaction does not result in any change
in
the working capital, it is said that it does not result in the flow of
fund.
Preparation of Fund Flow :– For preparing Fund Flow
Statement
we have to prepare the following three Statement
statements:
(1) Schedule of Changes in Working This schedule considers
current
Capitalassets
:– and current liabilities, at the beginning and at the end
only
of
the year. This schedule shows either increase or decrease in
working
capital
.
SCHEDULE OF CHANGES IN WORKING
CAPITAL
Particulars Amount Amount Increase in Decrease in
As on As on Working Working
……….. ……….. Capital Capital
Current Assets:
Ø Cash-in-hand
Ø Cash at Bank
Ø Debtors
Ø Closing Stock
Ø Short Term
investments
Ø Bills Receivables
Ø Prepaid Expenses
Ø Other current assets ________ ________

192

footer
ACCOUNTING FOR
MANAGERS
Total Current Assets
Current
Liabilities:
Ø Bank Overdraft
Ø Bills Payable
Ø Creditors
Ø Provision for Taxation
Ø Proposed Dividend
Ø O/s Expenses
Ø Unclaimed Dividend ________ ________ ________ ________

Total Current Liabilities ________ ________ ________ ________


Working Capital (Current
Assets –Current Liabilities)
Net Increase or Decrease
in Working Capital

(2) Calculation of Funds from In order to prepare a


Operations
flow statement it is:–necessary to ascertain the sources and application
funds
of
funds. Main source of fund in a business is funds from
operations
STATEMENT SHOWING ‘FUNDS FROM
OPERATIONS’
Particular Amount
s
Net Profit As per Profit & Loss
A/c
(A) Items to be Added back to Net
Profit:
(a) Non-Fund
Items:(i)
(ii) Goodwill written
Depreciation.
off.
(iii) Preliminary
Expenses.
(iv) Patent Rights, Trade Marks and Copy
Rights.
(v) Discount on issue of Debentures &
Shares.
(vi) Deferred Revenue Expenditure such
as, Advertisement Suspense
A/c.
(b) Non- Trading
Losses:(i) Loss on sale of Fixed
Assets
(ii) Appropriation of
Profit:Transfer to General
Reserve
Transfer to Sinking Fund
Transfer to
Dividend
Equalisation Fund
(i) etc. Proposed
Dividend

193

footer
(ii) Provision for
Taxation
(B) Items to be Deducted from Net
Profit(i) Profit on sale of Fixed
Assets
(ii) Receipt of
Dividend
(iii) Re-Transfer of Excess
Provisions
Ø Fund Flow Statement :– Fund Flow Statement is prepared to show
changes in assets, liabilities andthecapital between the dates of two
balance
sheets. It discloses the causes of changes in the items of balance
sheet
between the end of the previous year and the end of current year. Thus,
by
preparing this statement, the management can find out the basic
reasons
for changes in the assets, liabilities and capital of the firm between
two
balance
sheets.

FORMAT OF FUND FLOW STATEMENT :–

FUND FLOW STATEMENT

Sources of Funds Applications of


Funds
1. Funds from Operations 1. Loss from
Operations
2. Issue of Shares 2. Buy back of Equity shares
and Redemption of Preference
3. Issue of Debentures 3. Redemption Shares of
Debentures
4. Raising Long-Term Loans 4. Repayment of Long-Term
Loans
5. Sale of Fixed Assets 5. Purchase of Fixed
Assets
6. Non-Trading Receipts 6. Non-Trading
Payments

Q. What is Fund Flow Statement? What are the uses and Limitations
of Fund Flow Statement?
Ans. Meaning of Fund Flow :– The balance sheet of a firm
Statement
the position of assets, liabilities and capital at the enddiscloses
of a particular year.
But
it does not disclose the causes of changes in these items between the end
of
previous year and the end of current year. Therefore, an additional
statement
called ‘Fund Flow Statement’ is prepared to show the changes in
assets,
liabilities and capital between the dates of two balance
sheets.
Uses of Fund Flow :–
Statement

194

footer
ACCOUNTING FOR
MANAGERS
1. Helpful in Finding the Answers to Some Important
Financial
Questions : A fund flow statement is prepared to give satisfactory

to the answer
following
questions:
(i) What have been the main sources and applications of funds
duringthe
period?
(ii) How much funds have been generated from business
operations?
(iii) Where did the profits
go?
(iv) Why are dividends not
larger?
(v) How was the expansion in plant and Equipment
financed?
(vi) How was the repayment of long term debt
accomplished?
(vii) How was the increase in Working Capital
financed?
2. Helpful in Financial A fund flow statement provides
Analysis :–
complete analysis of the financial position of a firm. This objective is
not
achieved by the balance sheet because it gives a static view of the
financial
position of a business by showing the assets and liabilities at a
particular
point of time.
3. It provides more reliable figures of profit and loss of the A
business
fund flow:–statement gives a much more reliable figure of the profits of
the
business than the figures shown by profit and loss account is affected
by
the personal decisions of management in deciding the amount
of
depreciation and other adjustments regarding the writing off
of
preliminary expenses
etc.
4. It enables to know whether the funds have been properly The
used :–
fund flow statement enables the management to know whether the
funds
have been properly used in purchasing various assets or repaying
loans
etc
.
5. Helpful in proper Management of Working While
Capital :–capital in a business, it becomes essential to ensure that itmanaging
working
should
neither be excessive nor inadequate. A fund flow statement indicates
the
excessiveness or inadequacy in working
capital.
6. Help in the presentation of Budget for the next If a fund
period :– is prepared for next year, it will enable the management to
statement flow
plan
its financial resources properly. The firm will know how much funds
it
requires, how much the firm can manage internally and how much
it
should arrange from outside source. This is helpful in preparing
the
budgets for the future
period.
7. Helpful in Determining Dividend Sometimes, there may
Policy :– profits but the distribution may not be possible due
sufficient be to
its
adverse effect on the liquidity and working capital of the
business.
Limitations of Fund Flow
Statement :–
195

footer
1. Fund flow statement ignores certain non-fund
transactions.
2. It reveals only the changes in working capital and does not show
the changes in cash
3. It isposition.
historical in nature because it reports what has happened in the
past.
4. Since it is based on opening and closing balance sheets and the profit
and loss account, it is not an original
statement.
Q. What is Cash Flow Statement? Give the Format of Cash
Flow Statement Or How is it
prepared?
Ans. Cash
A cash-flow statement
Flow is :–a statement
showing
inflowsStatement
and outflows of cash during a particular period. In other words, it is
asummary of sources and applications of cash during a particular span of
It analyss the reason for changes in balance of cash between the two
time.
balance
sheet dates. The term ‘cash’ here stands for cash and cash equivalents. A
cash-
flow statement can be for the past or can be projected for a future
period.
FORMAT OF CASH FLOW STATEMENT :– A cash flow statement may
prepared either by direct or indirect method. Format underbeindirect method
is
given below:
XYZ LTD.
CASH FLOW STATEMENT for the year
ending……………… (Indirect
Method)

Particulars Amount
Amount
(A) Cash Flows from Operating
Activities :–
Net profit before
Tax .............
Add : Non-Cash .............
Expenses
: Non-operating .............
Expenses
Operating Profit before Working Capital Changes
________
Add : Decrease in Current .............
Assets: Increase in Current .............
Liabilities ________
Less : Increase in Current Assets .............)
( : Decrease in Current Liabilities .............)
( ________
Cash generated from operating .............
activities
Income Tax (.............)
paid

196

footer
ACCOUNTING FOR
MANAGERS
________
Cash flows before extraordinary .............
items
(+) or (-) Extraordinary .............
items
Net Cash from Operating Activities .............
________
(B) Cash Flows from Investing
Activities :–
Ø Purchase of fixed assets .............)
Ø (
Sale of fixed .............
Ø assets
Purchase of investments (long-term) .............)
Ø (
Sale of Investment (long- .............
Ø term)
Interest .............
Ø received
Dividend .............
Net Cashreceivedfrom Investing Activities .............
________
(C) Cash Flows from Financing
Activities:
Ø Proceeds from issue of share .............
Ø capital
Proceeds from long-term .............
Ø borrowings of long-term borrowings
Repayments .............)
Ø (
Interest paid .............)
Ø (
Dividend paid ( .............)

Net Cash from Financing Activities .............


_________
Net Increase or decrease in cash and ________
cash
equivalents .............
( A+B+C)
Cash and cash equivalent at the .............
of the period
beginning ________
Cash and Cash equivalent at the .............
end
of the ________
period
Q. What is Cash Flow Statement. Discuss Its main Uses
and Limitations
?A cash-flow statement
Ans. Cash Flow is :–a statement
showing
inflowsStatement
and outflows of cash during a particular period. In other words, it is
asummary of sources and applications of cash during a particular span of
time.
It analysis the reason for changes in balance of cash between the two
balance
sheet dates. The term ‘cash’ here stands for cash and cash equivalents. A
cash-
flow statement can be for the past or can be projected for a future
period.
Uses of Cash Flow :–
Statement
197

footer
1. Useful for short-term financial A cash flow
planning information
provides :– statement
for planning the short-term financial needs of
the
firm. Since it provides information regarding the sources and utilization
of
cash during a period, it becomes easier for the management to
assess
whether it will have adequate cash to meet day-to-day expenses and
pay
the creditors in
time.
2. Useful in preparing the Cash A cash flow statement
Budget
for :–
the future period in helpful in preparing a cash prepared
budget. It informs
the
management about the surplus or deficit periods of cash. It helps
in
planning the investment of surplus cash in short-term investments and
to
plan short-term credit in advance for deficit
period.
3. Study of the trend of cash receipts and A cash
payments reveals
statement :– the speed at which the cash is being generated flow
from
debtors, stock and other current assets and the speed at which
the
current liabilities are being paid. It enables the management to assess
the
true position of the cash in
nature.
4. It explains the deviation of cash from A firm may earn
earnings
profits yet:–it may have paucity of cash or when it suffered a losshugeit may
still
have plenty of cash. A cash flow statement explains the reasons for
it.
5. Helpful in making Dividend Dividend must be paid
Decisions
42 days of:–its declaration. Hence the management takes within
the help of
cash
flow statement to ascertain the position of cash generated from
operating
activities which can be used for payment of
dividend.
6. Study of the Trend of Cash Receipts and A cash-
Payments reveals
statement :– the speed at which the cash is being generated flow
from
debtors, stock and other current assets and the speed at which
the
current liabilities are being paid. It enables the management to assess
the
true position of the cash in
future.
Limitations of Cash Flow
Statement :–
1. It does not present true picture of the liquidity of a firm because
the
liquidity does not depend upon cash
alone.
2. The possibility of window-dressing is higher in case of cash position
in
comparison to the working capital position of a
firm.
3. Cash flow statement ignores non-cash
charges.
4. It is prepared on cash basis and hence ignores one of the basis concepts
of
accounting, namely accrual
concept.
Q. Distinction between Fund Flow Statement and Cash Flow
Statement.
Ans. Distinction between Fund Flow Statement and Cash Flow :
Statement
198

footer
ACCOUNTING FOR
MANAGERS
BASIS FUND FLOW CASH FLOW
STATEMENT STATEMENT

1. Basis of It discloses the causes of It discloses


Analysi
the changes in working capital. cause of
s changes in cash
position.
2. Interpretation Sound funds position doesn’t Sound cash
position mean sound cash position means sound
funds
because inflow of funds position as inflow
of
doesn’t necessarily involve cash
necessarily
inflow of cash. involves
inflow of funds.

3. Difference in In case of fund flow analysis, In case of cash


flow
the method of an increase in a current analysis
an
preparation liability or decrease in a increase
or current assets results in decrease in
adecrease in working capital current
assets
& vice-versa. results in
increase in cash
&
vice-
versa.
4. Usefulness It is more useful for long-term It is more useful
for financial planning. short-term
financial planning.
5. Schedule of Schedule of changes in No
separate
Changes in working capital is also schedule
is
Working Capital preparing to study the prepared with
the changes in current assets cash flow
statement
and current liabilities. as the changes
in current assets
and
current
liabilities
are shown by way of
adjustment in
profit
to arrive at
cash
flow from
operating
activities
.
6. Opening & Opening & closing balance Opening &
closing Balance of cash are not shown in fund balance of cash
are
of Cash flow statement, as they are shown in
this shown in schedule of changes
statement
in working
capital.

199

footer
7. Principles of This is prepared on ‘accrual This is prepared
on
Accounting basis’ of accounting ‘cash basis’
of accounting.

Q. Distinction Between Fund Flow Statement And Income Statement


Or Profit And Loss
Ans. Account.
Distinction Between Fund Flow Statement And Income Statement
ProfitOr And Loss
Account
BASIS FUND FLOW STATEMENT INCOME
STATEMENT

1. Nature It discloses the sources and It discloses


net uses of funds. profit or net
loss made during
the
year.
2. Items All items whether capital or Only revenue
items revenues are considered in are considered
in
preparing this statement. this
statement.
For Example:- Cash received For Example:-
Cash
from shares issue is received from
share
considered a source of funds issue is
not considered
an
income.

3. Objects It helps in the financial It is prepared


to management of the business ascertain the
net results of
business
operations
.
4. Optional or Preparation of fund flow Preparation
ofCompulsory statement is optional and if it income statement
is is prepared, it can be one so compulsory and
the
in any manner. contents of
income statement must
be
in accordance
with
requirements of
law.
5. Help Income statements help the Funds
flow preparation of funds flow statement
doesn’t
statement because funds from help
the
operations are found out from preparation
of
income statement. income
statement.

200

footer
ACCOUNTING FOR
MANAGERS
6. Scope Its scope is limited because it It is of
utmost only shows changes in important
because
working capital, i.e. sources it depicts the
result
and uses of funds. of all
financial transactions of
aparticular
period.
Q. Distinction Between Fund Flow Statement And Balance
Sheet.
Ans. Distinction Between Fund Flow Statement And Balance :
sheet

BASIS FUND FLOW STATEMENT BALANCE SHEET

1. Nature It discloses the sources and It is a statement


of uses of funds assets and
liabilities at appoint of
time.
2. Items All items whether capital or Only the items
of revenues are considered in assets,
liabilities
preparing this statement. and capital
are considered
in
preparing
this
statement
.
3. Use It is a tool of management Not only
the techniques and is used by the management
but
management for future some
external
financial planning and budget parties are
also
formation. interested in
it because it
is
prepared to
reveal
the
financial
position of the
firm.
4.Optional or Preparation and publication of Preparation
and
Compulsory fund flow statement is optional publication
of and if it is prepared, it can be Balance Sheet
is
done in so in any form. compulsory and
it is prepared in
its
prescribed
form.

201

footer
5. Basis of It is prepared with the help of It is prepared
with
preparation two balance sheet and profit & the help of
trial loss A/c.
balance.
6. Scope Its scope is limited because it It is so
important only shows changes in because it
shows
working capital, i.e. sources the
financial
and uses of funds. position of
the business
.
7. Term It is prepared to know the It is an yearly
affair temporary changes throughout
the whole life of
the
business
.

202

footer
ACCOUNTING FOR
MANAGERS
MBA 1st Semester
(DDE)

UNIT – III

Q. What is Management Accounting? Explain its Nature,


Importance
and Roles.
Ans. Introductio
The main objective:– of financial accounting is to
providen
information about the profitability and financial position of an enterprise
by
preparing trading and profit & loss account and a balance sheet. But it does
not
present the accounting information in such a way to assist the management
in
planning day-to-day operations of a business and to make various types
of
decisions. There are various limitations of financial accounting
and
management accounting removes these
limitations.
Meaning of Management :– Management Accounting
Accounting
comprises of two words ‘Management’ and ‘Accounting’. Itisis the study
of
managerial aspect of accounting. The emphasis of management accounting
is
to redesign accounting in such a way that it is helpful to the management
in
formation of policy, control of execution and appreciation of effectiveness. It
is
that system of accounting which helps management in carrying out
its
functions more efficiently. Management Accounting presents the financial
data
in such a way as to assist the management in planning and controlling
the
activities of the firm. Management Accounting is also known as ‘accounting
for
management’
.
Definition of :–
Accounting
According to Robert. N.
Anthony“Management Accounting is concerned with accounting information
is thatuseful to
management”.
According to Institute of Chartered Accountants of England and
Wales“ Any form of accounting which enables a business to be conducted
more can be regarded as management
efficiently
accounting”.
Nature of Management :–
Accounting
1. It lays more Emphasis on Management accounting
Future :– with the future. It helps the management inisforecasting
concerned
and
planning the future course of
action.
203

footer
2. Techniques of selective It is a technique of selective Nature.
Natureinto
takes :– consideration only that data from It the profit and loss
account
and balance sheet which is relevant and useful to the management.
Only
that information is communicated to the management which is helpful
to
decision
making.
3. It establishes cause and effect The cause and
Relationships
relationship is :–
studied in management accounting. For Example,effect if
the
profits are lower than expectations, the reason for the same
are
investigated. On the other hand, if the profits are more than
expectations,
reasons for the higher profitability are analysed. The effect of
various
decisions such as pricing, promoting a new product, sales mix,
cost
control etc. is studied on the profitability of the
business.
4. It Provides Information and not the The
Decisions :–never takes any decisions but only provides data on themanagement
accountant
basis
of which the management takes
decisions.
5. Use of Special Techniques and The
Concepts :– uses various techniques and concepts to make the
accountant management
accounting
data more useful for managerial decision
making.
6. No Set Rules and No specific rules are followed
Formats
management :– accounting. It provides information
in in the form which
may
be more suitable to the management in taking various
decisions.
Importance of Management :– Management accounting is
Accounting
important because it enables management to maximize very profits or
minimize
losses. Its importance is as
follows:-
1. Planning and Policy Formation :– Management accounting
information to the management for formulating plans. supplyPlanning
is
essentially related to taking decisions for future. It also
includes
forecasting setting goals and deciding alternative courses of action.
So
management Accounting is helpful in planning and policy
formation.
2. Helpful in controlling Management accounting
performance
like standard :–costing & budgeting controls are devices
helpful in
controlling
performance. The work is divided in to different units & separate goals
are
set up for each unit. The management accounting act as a co.
ordinating
link between different departments and he also monitors the
performance
of top
management.
3. Organisation
Helpful is related to the in establishment
of Organizing
relationship :–among different individuals in the concern. It also
includes
delegating of authority and fixing of responsibility. All these aspects
are
set up by management
accounting.
204

footer
ACCOUNTING FOR
MANAGERS
4. Helpful in interpreting financial the main object
information :–
management accounting is present financial information to themofin
such
away that it is easily understood. Management accounting explains all
the
statement to the management in simple language. If necessary he
uses
statistical devices like charts, diagrams, index number etc. so that
the
information easily
followed.
5. Helpful in Management accounting he lps
Coordination the
coordinating :– activities of differentin persons and different
departments
by establishing a ‘management information
system’
6. Management
Motivatingacempl counting
oyees helps
:
the –management in selecting best alternatives of doing the things. Targets
are
laid down for the employees. They feel motivated in achieving their
targets
and further incentives may be given for improving their
performance.
7. Helpful in making The management has to take
decisions :–decisions. A decision may have
important certain
to be taken about the
expansion
or diversification of production. The information provided by
the
accounting helps the management in selecting a suitable alternative
and
taking correct
decision.
8. Reporting to One of the primary importances
management :–
management accounting of
is to keep the management fully inform
about
the latest information of the concern. This helps the management
taking
proper & timely decisions. Management accounting is a new approach
to
accounting. It provides techniques for the interpretation of
accounting
data. The following facts of management accounting are of a
great
significance and form the scope of this
project
9. Management
Increases accounting helps to increase
the Efficiency
efficiency of:– management. The targets of various departments are
fixed
and communicated to them well in advance. This helps in increasing
the
efficiency of the
management.
10. Management
Maximizing accounting measures
the Profitability :– each activity and each department. Only those
profitability of
activities
and departments are continued which promise better
results.
Unprofitable activities and departments are discontinued. It
maximizes
the profitability of the
organization.
Role of Management The functions of
Accounting depend
accountant :– upon his status in the organisation,management
needs of the
enterprise
and personal capabilities of the persons. So role of management is
very
necessary for every organisation. Its role is as
follows:-
1. Management
Planning Accountant for establishes,
co- control :–
205

footer
ordinates and maintains an integrated plan for the control of
operations.
Such a plan would provide cost standards, expenses budgets,
sales
forecasts
etc.
2. Management
Reporting :–accounting measures performance
againstgiven plans and standards. The result of operations is interpreted to
all
levels of management. This function will include installation of
accounting
& costing systems and recording of actual performance so as to find
deviation
out (if
any).
3. Management
Evaluating :– accountant should evaluate various
policies
and programmes. The effectiveness of planning and procedures to
attain
the objectives of the organisation will depend upon the caliber of
the
management
accountant.
4. Management
Administration
accountant of
is expected to
report to
Tax :– agencies as required under different laws and to supervise
govt.
all
matters relating to
taxes.
5. Appraisal of external He is to assess the effective
effects :– and fiscal policies of the govt. andvarious
economic also evaluate the impact
of
other external factors on the attainment of organisational
objects.
6. The Protection
protection of business
of assets
anotherAssets
function:–assigned to the management accountant. This function
is
performed through the maintenance of internal control, auditing
and
assuring proper insurance courage of
assess.
Q. Explain Man agemen t Accountin g. Des crib e th e S cope
Of Management
Accounting.
Ans. Meaning of Management :– Management Accounting
Accounting
comprises of two words ‘Management’ and ‘Accounting’. It isisthe study
managerial aspect of accounting. The emphasis of management accounting
of
is
to redesign accounting in such a way that it is helpful to the management
in
formation of policy, control of execution and appreciation of effectiveness. It
is
that system of accounting which helps management in carrying out
its
functions more efficiently. Management Accounting presents the financial
data
in such a way as to assist the management in planning and controlling
the
activities of the firm. Management Accounting is also known as ‘accounting
for
management’
.
Definition of
Accounting :–
“Management Accounting is concerned with accounting information
is thatuseful to Robert. N. Anthony
management”.
Scope of Management :– Scope of management
includes all those activities which are helpful in the collection
Accounting accountingand analysis
of
information
.
206

footer
ACCOUNTING FOR
MANAGERS
Scope of Management
Accounting

Internal Interpretation
Audit of data

Financial Inventory
Accounting Cost Control
Tax
Accounting Financial Accounting
Budgeting &
Management Forecasting

1. Financial
Financialaccounting deals with the
historical
accounting
data :–
the recorded fact about an organisation are use for planning
the
future course of action. Though planning is always for future but still
it
has to be based on past and present data. The control aspect too is
on financial data. The performance appraisal is based on recorded
based
facts
and figures. So management accounting is closely related to
financial
accounting.
2. costCost
accounting provides various techniques
for Accounting
determining :–cost of manufacturing products or cost of providing
service.
It uses financial data for finding out cost of various jobs, products
or
process. The system of standard costing, marginal costing etc. are
all
helpful to management for planning various business
3. It activities.
Financial
is concerned with planning and
controlling
Management
of the financial:–resources of a firm. It deals with raising of funds and
their
effective utilization. Its main aim is to use business funds in such a
way
that earnings are
4. maximized.
Budgeting and Budgeting means expressing the
Forecasting
policies and :–
goals of the enterprise for aplans,
definite period in future.
The
targets are set for different departments and responsibility is fixed
for
achieving these targets. The comparison of actual performance
with
budgeted figures will give an idea to the management about
the
performance of different departments. Forecasting also helps
the
management accountant in planning various
activities.
5. Inventory
Inventory is used to denote stock of raw
materials,
Controlin:– process and finished goods. Inventory has a special
goods
significance
in accounting for determining correct income for a given
Management will need effective inventory control for controlling
period.
stocks
and this will helpful for taking managerial
decisions.
6. TheInterpretation
management accountant
of
interprets
Data
various:– financial statements to the management. These statements
an
giveidea about the financial and earning position of the concern. If
the
207

footer
statements are not properly interpreted, these wrong conclusions may
be
drawn. So interpretation is as important as compiling of
financial
statements
.Internal
7. Internal Audit system is necessary to judge
the Audit :–
performance of every department. The actual performance of
every
department and individual is compared with pre-determined
standards.
Internal audit helps management in fixing responsibility of
individual.
different
8. Tax Tax planning
Accounting is :– an important part of
management
accounting. Income statements are prepared and tax liabilities
calculated
are
.
Q. What are the Functions or Objectives and Limitations
of Management
Accounting?
Ans. Meaning of Management :– Management Accounting
Accounting
comprises of two words ‘Management’ and ‘Accounting’. It isisthe study
managerial aspect of accounting. The emphasis of management accounting
of
is
to redesign accounting in such a way that it is helpful to the management
in
formation of policy, control of execution and appreciation of effectiveness. It
is
that system of accounting which helps management in carrying out
its
functions more efficiently. Management Accounting presents the financial
in such a way as to assist the management in planning and controlling
data
the
activities of the firm. Management Accounting is also known as ‘accounting
for
management’
.Definition of :–
Accounting
“Management Accounting is concerned with accounting information
is thatuseful to Robert. N. Anthony
management”.
Functions or Objectives of Management :– The basic function
Accounting accounting is to present the information to the management
management of
in
such a way that it is helpful to the management in taking correct decisions.
main functions of the management accounting
The
are:-
1. TheCollection
first function of management
of accounting is
the collection
Data :– of data which is useful to the management. The data is
collected
from internal as well as external sources. Internal sources include profit
&
loss account, balance sheet, cost records, sales reports etc.
External
sources include business magazines and publication of
government
bodies
etc.
2. After
Modification
collection of data, itofhas to be modified
in Data a:–way that it becomes useful to the management. For example
such
the
production data is classified on the basis of product, quality, time taken
by
manufacturing process etc. Data is modified according to the purpose
which
for the data is
required.
208

footer
ACCOUNTING FOR
MANAGERS
3. Analysis and Interpretation of Financial One of the
Statementsfunctions
significant :– of management accounting is the analysis most
interpretation of profit & loss account and balance sheet of the
and
enterprise.
These are analysed to show the position of liquidity, profitability
and
solvency of the
4. enterprise.
Use of Qualitative information Management accounting is
also :– to the use of quantitative information only.
restricted not It collects and
makes
use of qualitative information
also.
5. Planning
To helpis ina planning
basic function
:– of management
and management accounting provides necessary information to
the
management for formulating
plans.
6. Organization
To help in Organizing
is the division
:– of
duties,authorities and responsibilities among various persons working in
enterprise. Management accounting collects necessary data about
the
the
performance of each individual and department and on this
basis
evaluates their performance and fixes the
responsibility.
7. Management
To Help in Control
accounting
:– techniques like
budgetary
control and standard costing are very helpful in the control process.
In
management accounting separate standards are fixed for
department and actual results are compared with the
each
standards.
Variances are ascertained and taking corrective
actions.
8. Helpful in Management accounting he lps
coordinating
Coordination the
:– activities of differentin persons and different
departments
by establishing a ‘management information
9. system’
To help in Decision- Management has to make
Making :– These decisions pertain to various
decisions. introduction of latest
technological
devices, selection of product mix, dividend policy etc.
Management
accounting helps the management in selecting a suitable alternative
in
and taking the best possible
decision.
10. Communicating Up-to-date One of the main objectives
Information
management :– of
accounting is to provide latest information to
managers.
various Information is provided through various
reports.
Limitations of Management :– Although
accounting
Accounting has improved the efficiency of management by providing
management
various
service, it suffers from the following
limitations:-
1. Limitation of Financial Accounting and Cost
Accounting :–accounting mainly uses the data provided by financial
Management
and
cost accounting. These data are historical in nature and are based
on
certain accounting assumptions and conventions. Hence, all
limitations of financial accounting and cost accounting enter
the
in
management
accounting.
209

footer
2. Lack of Knowledge of Related The use of
Subjects :– requires the knowledge of a number of relatedmanagement
accounting subjects
such
as accounting, statistics, principles of management and so on. It is
very
difficult for the person who is taking the decisions to have a
proper
knowledge of all these
subjects.
3. Effected by Personal Personal views are involved in all
Views :– right from the collection of information
activities, the till the preparation
of
reports submitted to the management. Different persons can
draw
different conclusions from the same information. Hence, there is a
scope
for bias in management
accounting.
4. The Costly
installation of management accounting
systemrequires
System :–a large organization and a wide network of rules and
regulations
and hence requires a heavy investment. Therefore, smaller
organizations
cannot afford
it.
5. Management
Evolutionaryaccounting
Stage :– is still passing
through its evolutionary stages and has not yet developed
fully.
6. Not an alternative to Management accounting is not
Management :–
alternative to management. It provides only the an
informations and
not
decisions
.
Q. What are the tools and techniques used in management
accounting?
Ans. Management accounting is not a separate method in itself. It is
acombination of various tools and techniques as
follows:-
(1) Ratio
Analysis
(2) Funds Flow
Analysis
(3) Cash Flow
Analysis
(4) Other techniques of analysis of financial statements such as
trend analysis and comparative financial
(5) statementsBudgetary
Control
(6) Standard
Costing
(7) Marginal
Costing
(8) Communicating Or
Reporting
(9) Accounting for Price Level
Changes
(10) Human Resource
Accounting.
Q. Distinguish Between Cost Accounting and Management
Accounting.
Ans. cost
Costaccounting provides various techniques
for Accounting
determining cost of:–manufacturing products or cost of providing service.
It
uses financial data for finding out cost of various jobs, products or process.
The
system of standard costing, marginal costing etc. are all helpful to
management
for planning various business
activities.
210

footer
ACCOUNTING FOR
MANAGERS
Management Management Accounting presents the
Accounting
data in such :– financial in planning and controlling
a way as to assist the management
activities of the firm. Management Accounting is also known as ‘accounting
the
for
management’
.
Distinguish between Cost Accounting and Management
Accounting:
Sr. Basis of Cost Accounting Management
Accounting
No.
Difference
1. Objects The main objects of cost The main objects
of accounting are the management
accounting
ascertainment of cost, is to provide all types
of
controlling the cost and information
providing the cost data managerial
for
decision
to the management. making, whether
derived from financial accounts
or
from cost accounts
or
from any other
source.
2. Scope The scope of cost Scope of
management accounting is narrow accounting is wider
than
because cost accounting cost
accounting.
provides information Management
accounting
about cost only. includes in its
scope financial accounting,
cost
accounting, budgeting,
reporting
etc.
3. Nature of Cost accounting is Management
accounting
data used related to both past as is concerned only
with well as future. It the future because
maintains
it the cost provides information
for
records on historical decision making
and
basis and also makes planning for the
future.
estimates for the
future.
4. Use of Cost accounting follows Management
Principles certain principles for does not follow
accounting
set recording, classifying principles. It may
provide
and ascertaining the information in
any
costs of products. manner suitable
for decision
making.
5. Use of In cost accounting only Management
accounting
Qualitative that data is considered uses both quantitative
as
Data which can be expressed well as qualitative
data.
211

footer
in quantitative terms, Qualitative data
means non-monetary events
like
technological
innovation,
change in
management,
customer
satisfaction,
competition
etc.
6 Audit Cost audit has been No audit is required
for made compulsory in management
accounting.
certain
companies
specified
under
section 233 B of
the
Companies Act, 1956.

Q. Distinguish between Financial Accounting and


Management
Accounting.
Ans. Management
Management Accounting presents :–
the Accounting
financial data in such a way as to assist the management in planning
and
controlling the activities of the firm. Management Accounting is also known
as
‘accounting for
management’.
Financial accounting :– Financial accounting deals with the
historicalare use for planning the
data the recorded fact about an organisation
future
course of action. Though planning is always for future but still it has to
be
based on past and present data. The control aspect too is based on
financial
data. The performance appraisal is based on recorded facts and figures.
So
management accounting is closely related to financial
accounting.
Distinguish between Financial Accounting
and Management
Accounting:
Sr. Basis of Financial
Management
No. Accounting
Difference Accounting
1. Objects The main objective of The main objective
of financial accounting is The main
objective
to keep a systematic ofmanagement
accounting
record of the is to assist in the
internal
transactions of a management of
an
business and the enterprise by
providing
preparation of profit the
necessary
and loss account and information
for
the balance sheet. decision
making.

212

footer
ACCOUNTING FOR
MANAGERS
2. Nature of Financial accounting is Management
accounting
Data Used concerned with the is concerned with
future. historical records
and
shows only the
past
results of a
business.
3. Subject Financial accounting Management
accounting
Matter measures the measures the
profitability profitability of the entire of different
products,
business as a whole. activities, departments
or units
separately.
4. Legal Financial accounting is Management
accounting
Compulsion compulsory for every is
optional. business because
of
legal
provisions.
5. Accounting Financial accounts are Management
accounting
Principles prepared on the basis of does not follow
set accounting principles
principles.
and it has to
follow
double entry
system.
6. Periodicity Financial statements There is no definite
period are prepared usually for for
preparing
one year. management
accounts.
7. Accuracy In Financial Accounting In
management only actual figures are accounting since
the
recorded. information is needed
as soon as possible,
the
approximated
figures
which are
quickly
available are
considered
more useful in
comparison
to the accurate
figures
which will be
available
too
late.
8. Monetary Financial accounting In
management
Transactions records only those accounting both
monetary transactions which can and non-
monetary
be measured in terms information are
of money.
used.

213

footer
9. Publications Financial accounts like Management accounts
are profit & loss account prepared for the
internal
and balance sheet are use of the
management
published for the use of only and hence these
are
general public. not
published.
10. Audit Financial accounts can Management
accounts be audited and in case cannot be audited
since
of companies the they are not based
in
financial accounts are actual
figures.
required to be
audited
by
Chartered
Accountant
s
Q. Explain the meaning of cost accounting and its importance
and contribution in
management.
Ans. Cost
Cost accounting is the :–
classifying, recording
and Accounting
appropriate allocation of expenditure for the determination of the cost
of
products or services and for the presentation of suitably arranged data for
the
purpose of control of management. Cost accounting includes the calculation
of
cost of every product, job, section, department, process, service
etc.
Cost accounting is the method of accounting forecast. Cost comprises
three
elements viz. material, labour and
expense.
Definition of Cost :–
Accounting
According to Wheldon,
“Cost Accounting is the application of accounting and costing
principles,
methods and techniques in the ascertainment of costs and the analysis
of
variances as compared with standards or previous
experience.”
Feature of Cost :–
Accounting
1. It provides information for
cost.
2. It records income and expenditure relating to production of goods
and services
3. It .helps to prepare tenders and quotation
prices.
4. It helps in cost
control.
5. It helps in preparing budgets and standard of cost and the
variance
analysis for controlling
6. Costcost.accounting helps in management in planning, control and
decision-
making.
Importance of Cost :–
Accounting
1. Cost
Cost
knowing or cost finding is
cost Ascertainment :–
214

footer
ACCOUNTING FOR
MANAGERS
ascertainment. This is the primary objective of cost accounting.
The
technique of ascertaining cost is known as costing. With the help of
cost
accounting, cost per unit of a product is
calculated.
2. Cost
Cost
control is one of the important function of
cost accounting.
Control :– Cost control is reducing the cost of production by
controlling
the wastage of material, labour and other expenses. Cost control helps
improve the efficiency of organization as a whole. For cost
to
control
budgetary control, standard costing are the main tools applied by
the
management
.
3. Ascertainment of It is the object of cost accounting
Profitability
ascertain the :– to
profitability of the activities carried out. Profitability
means
capacity to earn profits of a department, section, and product.
This
capacity can be judged with the help of cost data provided by
cost
accounting. The expected profits are compared with the actual profits
to
know the reasons of difference, if there are any. This helps to analyse
the
efficiency of each segment of the organization as well as for the whole of
organization
the
.
4. Determination of Selling The supply price or a tender price of
Price :– depends upon its total cost. Cost
product a accounting provides
detailed
information about the composition of total cost for the determination
of
the selling
5. price.
Providing a basis for business Cost accounting helps to
policy
accurate:– cost information. These cost data help provide
the management in
taking
short term and long term business policies to be followed. Cost
accounts
help in various decision making processes which may be very crucial
to
the
organization.
6. TheInternal
objective of costAudit
accounting is to
develop
System :–audit system which may help in effective working of
internal
departments
different of the
organization.
7. Effective Information The objective of cost accounting is
System :–regular reports regarding material,
prepare to labour and other
expenses
and to communicate those reports at the effective level of management
to
make them
effective.
8. Cost
Inter-firm
accounting helps in making
comparison
Comparison
of cost or of :–
profits of one firms with other firm operating in the
same
industry.
Contribution of Cost Accounting to :–
Management
1. Identification of Profitable and unprofitable activities of
the
Organisation :– Cost accounting provides a system where per unit cost
is management can easily distinguish those
calculated. Thus, the
products
215

footer
or departments which are earning profits or those running into low
profits
or
losses.
2. Calculation of Quotation Cost of production can be
Price
on the:–basis of actual information as well ascalculated
can be estimated on the
basis
of past performance. This helps to quote the price of tenders
without
actually performing
job.
3. Helps in formulating Business Cost accounting helps
Policies :–accurate cost information. These cost datato help the
provide
management
in taking short term and long term business policies to be followed.
Cost
accounts help in various decision making processes which may be
very
crucial to the
organization.
4. Helps the management in Decision Making Managemen
Process
has :– vital decision which affects the present and future working
to take t
of
the organization like make or to buy decision, change in the method
of
production, fixation of selling price. All these need various types of
cost
data which are regularly supplied by cost
department.
5. Helps in Checking Material There are various techniques
Cost :– the wastage of material at the time of purchasing,
checking of storing
and
use of material by various production departments. For this purpose,
the
techniques like level of material, ABC analysis etc. are frequently
used.
6. It helps in making Cost accounts when are
comparison :– then these may help intra-comparison
systematically maintained
and inter-
firm
comparison regarding cost of production, profitability, fixing of
selling
price
etc.
7. Other Advantage :–
(i) Constant efforts are made to reduce the cost of production
throughoperation, research
techniques.
(ii) The proper utilization of plants and machines to the full
desiredcapacity is measured and wastage
controlled.
(iii) It helps the management to decide the investment
policy.
Q. What are the elements of cost. Give the classification of
Costs.
Ans. Cost
Elements
has basically
of three
:– elements, these
are:- Cost
1. Material
Cost
2. Labour Cost
3. Other
Expenses

216

footer
ACCOUNTING FOR
MANAGERS
Elements of
Cost

Material Labour
Expenses

Direct Indirect Direct Indirect Direct


Indirect
Material Material Labour Labour Expenses
Expenses
1. TheMaterial
material may be defined as the item from
whichproducts
Cost :– are
manufactured.
(i) Direct
Direct
materials are those materials which
are Material :–
either specially purchased for the production of product or which
are
visible in the final product, as leather in the shoes, wood in
furniture,
cloth in dress
etc.
(ii) These
Indirect
material do not form part of the
product.Materialmaterial
Indirect :– is not visible in the final product as lubricant
in
machine, glue in book binding
etc.
2. TheLabour
term labour
Cost may
:– be defined as the human efforts
by which materials are converted into finished
products.
(i) Direct
Directlabour is the labour which is
personally
Labour :–
engaged in the production of goods, running of machines. It is
also
known as direct
wages.
(ii) Those
Indirect
workers who are not directly engaged in
the production
Labour :– or running the machines but providing services or help
to
those who are operating the machines as storekeeper,
watchman,
cleaner, waterman
etc.
3. TheExpenses
expenses are
: incurred for producing a product
or –providing a service in addition to material and wages as rent of
factory,
insurance , telephone bill,
etc.
(i) Direct
Direct
expenses are those expenses which
are directly
Expensesidentified
:– with a particular job, product or
operation.
Direct expenses are also known as process expenses,
productive
expenses or prime cost
expenses.
(ii) Indirect
Indirect
expenses are those which cannot
be Expenses
identified :–with a specific job or
process.

217

footer
Classification of Classification of cost means grouping of
Cost :– to their common characteristics.
according cost The main classification of cost
are
as
follows:-
1. According to Nature or
Elements.
2. According to the Function of
Organisation
3. According to
Controllability
4. According to
Normality
5. According to Accounting
Period
6. Others
1. According
According
to this classification,
to the costs
are divided
Nature :– into three
categories.
(i) Material
(ii)
Cost Labour
Cost
(iii) Other
Expenses.
2. According
Accordingto this classification
to , the costs
are divided
Function :– into four
categories:-
(i) Manufacturing
In any product producing
organization,
Costs :– is the first activity which generates all other activities
production
of
the organization. It starts with purchase and supply of raw
material
to production deptt. and ends with the finished goods kept in
store.
(ii) These
Administrative
are the cost incurred for running
the Cost
office:–of the organization where planning and decision-making
are
undertake n. The se expenses are incurred on the
general
management and administration of the
(iii organization.
Selling and Distribution The production of goods has no
) Cost :– until and unless there is no demand by the customer.
meaning
To
create the demand, there is a need of selling and
distribution
overheads
.
(iv) Research and Development Research cost are those
Cost :–are incurred for searching new methodscosts
which of production or
new
product, new material so that it may attract new customers. For
all
this, the organization needs better laboratories, specialists staff.
This
involves heavy
3. Under expenditure.
According
this category, costs can
to be
divided
Controllability
into two :–
parts:-
(i) Controllable
Controllable cost includes those
expensesCost :–can be controlled such as wastage of material, wastage
which
of
power or fuel etc. Generally variable cost are
controllable
(ii) It Uncontrollable
includes those expenses which cannot
be controlled
Cost :– . Generally, fixed costs are
uncontrollable.
218

footer
ACCOUNTING FOR
MANAGERS
4. As According
per this classification, thetocost can
be Normality
divided :–
into:-
(i) Those
Normalcost which are expected to incurred on
the production
Cost :– of a product in advance before th production takes
place
are normal
cost.
(ii) TheAbnormal
cost which
Costis:–incurred in the abnormal
cases or unexpectedly or the cost which was not calculated in advance
but
takes place during the production process due to various reasons
like
power failure, accident, fire
etc.
5. According to Accounting According to this classification,
Periodbe:– divided into two
can cost
parts :–
(i) Capital
CapitalCosts
Cost
are:–those expenditure the benefits
of which are to be taken over a long enough future period.
These
expenditures are very heavy and are to be decided by the top level
of
management
.
(ii) Those
Revenueexpenditure which are incurred for day to
day running
Cost :– of the factory. For example purchase of material, payment
to
labour, payment of power bill
etc.
6. Others :

(i) Standard
Standard costCost
is the
:– pre-determined cost for
each element of production comparing the actual with the standard
and
recording the variance, if there is any, for analysis and
corrective
action
.
(ii) Marginal
Marginal cost may be defined as the change in
the total
Cost cost
:– due to increase or decrease in production by one unit.
It
means only the variable costs are considered as the marginal
cost.
(iii) ItReplacement
is the current market cost of replacing
an Cost
asset :–
.
(iv) Opportunity
Opportunitycost is the value of sacrifice
made or Cost :– on opportunity foregone in accepting the next
benefit
best
alternative
.
Q. What are the Techniques of
Costing?
Ans. Techniques
There are certain of techniques
:– for
ascertaining
cost Costing
which are used by the management in taking managerial decision
also.
The major techniques are as
follows:-
(1) AsHistorical
the name suggest, under this method first
the Costing
expenditure :– is incurred and then it is recorded and accounted
for
calculating cost of production per unit and cost of the product or
service.
219

footer
But this method is like a post-mortem of cost of production. This
method
cannot applied for measuring efficiency of the
organization.
(2) When
Uniform
several firms in the same industry adopt
the same
Costing :–
principles of cost accounting for comparison, it is known as
uniform
costing.
(3) Marginal
Marginal cost may be defined as the change in
the Costing
total cost:–due to increase or decrease in production by one unit. It
means
only the variable costs are considered as the marginal
cost.
(4) Under
Absorption
this system, both variable and fixed cost
are charged
Costing :– to the cost center. Under this technique, both fixed and
variable
costs are allocated to the
product.
(5) Under
Directthis techniques, all costs which are direct to
the Costing :–
product, process whether fixed or variable are charged to cost
center.
(6) It Incremental
is a technique of cost accounting which
studiesthe
Costing
change:– in cost and change in revenue due to change in level of
output.
The additional cost and additional revenue are analysed for
making
decision by the
management.
(7) Standard
Standard cost
Costing
is the :–pre-determined cost for
each element of production comparing the actual with the standard
and
recording the variance, if there is any, for analysis and corrective
Q. Whataction. are the Methods of Costing OR Cost
Accounting?
Ans. Methods
Various methods Of and techniques
:– have
been Costing
developed for cost accounting to meet the specific needs of the
business
organizations. The methods for calculating cost of production differ
from
industry. Basically there are two methods of costing. These
are: A.) Specific Order
costing
B.) Operation
Costing
Methods of
Costing

Specific Order Costing Operation


Costing
A. These
Specific
methods areOrder
applied where:–the work
or
job is costing
of a special nature as the design of the grill, design of house etc.
Under
this method production take place when the order is received from
the
customer. These
include :

220

footer
ACCOUNTING FOR
MANAGERS
1. JobsJobareCosting
to be performed
:– under the specific instructions
of the individual customer. Every customer has special taste, design,
quality
of work and spending power. So the jobs are not comparable to each
other.
Each job has its own unique features. Job are to be performed in a
small
duration, with in the factory premises, and they are generally of
small
value. Under job costing, work starts only after receiving the order of
customer e.g. printing press, tailoring of dress, interior
the
decorators,
machine tolls manufacturing
etc.
2. WhenBatchtheCosting
small :–orders of a large number of
customers
are made into one group on the basis if similarities if material, method,
or
nature of working, it becomes one batch of product to be produced.
A
batch may refer to a number of small orders passed through the factory
in
a Batch. The cost of each batch is calculated separately and one batch
is
considered as one cost
3. Thisunit.
Contract
method is applied in those cases where the
work Costing :–
to be performed in big involving lot of investment and take more than
one
year to be completed. Contracts are carried outside the factory,
generally
at the work site. In contract costing, cost of each contract is
ascertained
separately. The contracts are generally of construction nature
like
construction of roads, building, bridges, water work dams
etc.
B. When
Operating
goods are produced on mass scale and of
the Costing :– nature without waiting for the order of the customer.
standardized
The
produced goods are kept in stock and the orders of the customers
are
complied from the stock of finished goods. The production takes
place
continuously. This includes the methods
like :
(1) When
Processthe raw material
Costing
has to pass
through
Methodprocesses,
various :– or departments or stages before it is converted
into
finished goods, the method applies for the cost calculation in
these
industries is known as process costing system. The production
is
continuous under this method. In process costing, the output of the
first
process becomes the raw material for the second process and the output
the second process becomes the raw material for the third process and
of
so
on until the product becomes finished goods ready for sale. In
process
costing, the cost for production for each unit is calculated at the end
of
each process or
department.
(2) Unit Costing System or Single output This method of
Costing
is applied:–in those industries which produce identical output andcosting
only
one
product. The output takes place by continuous operating activities.
Cost
per unit is calculated by dividing the total cost by the number of
units
produced. Under this method, to calculate the cost of production, a
cost
sheet with all the components of cost is prepared for a specific period
or
specific order of
output.
221

footer
(3) In Operating
cost accounting cost per
Costing
unit of
a System
product :–
produced or a service provided is calculated. When cost per
unit
of service provided is calculated then the system of costing applied
is
known as operating
costing.Inventory. What are the objectives of Inventory
Q. Define
Valuation?
Ans. Every
Inventory
enterprise
:– needs inventory for smooth running of
its
activities. The term inventory refers to stock of goods kept for sale by the
firm.
Kinds of
Inventories:-
(A) In Trading Concern. (B) In Manufacturing
Concern.

(A) In Trading Concern :– In case of trading concerns, it includes only


finished
goods.
(B) InManufacturing
case of manufacturing concern,
inventory
Concern :–
may
include:-
(i) RawInventory
Material formofa majorRaw input
into the
Materials :–
organisation. The inventory of raw materials contains the
items
which are to be converted into finished goods through
the
manufacturing process. The quantity of raw materials required
be determined by the rate of consumption. The factors like
will
the
availability of raw materials and government regulations, etc.
too
affect the stock of raw
materials.
(ii Inventory of Work-in- The work-in-progress is that
) progress
of stocks :– stageand finished goods.
which are in between raw materials
The
raw materials enter the process of manufacture but they are yet
to
attain a final shape of finished
goods.
(iii) These
Inventory
are the goods
of which
Finished
are
ready for
Goodsthe :–consumers. In other words, inventory of finished
goods
represents completed items which are available for
Objectives sale. of Inventory :–
Valuation
1. TheDetermination
valuation of inventory isofnecessary
to determine
Income :– the true income earned by a business during a
particular
period. Gross profit is the excess of sales over cost of goods sold
is
ascertained by adding opening inventory to and deducting
closing
inventory form
purchase.
2. Determination of Financial The inventory at the end of
Position
period is :–
to be shown as a current asset in the balancea sheet of
the
business
.
Q. What are the methods for Valuation of
Inventories?
222

footer
ACCOUNTING FOR
MANAGERS
Ans. The
Valuation
value of materials hasofa direct bearing
on income
the Inventories :–
of a concern, so it is necessary that a method of pricing
materials
should be such that it gives a realistic value of stocks. The traditional method
of
valuing materials ‘Cost price or market price whichever is less’ is no longer
the
only method.
The following methods for pricing materials issues are generally
used :–(1) First in First Out Method (Known as FIFO
(2) Last in First Out Method (Known as LIFO
Method)
Method).
(3) Highest in First Out (Known as HIFO
Method).
(4) Average Price
Method.
(5) Base Stock
Method.
(6) Standard Price
Method.
(7) Market Price
Method.
(1) First in First Out (FIFO) In first in first out method
Method
materials:–received first are issue first. The materials
the are
issued in
chronological order. The recently received materials remain in
stock.
Whenever a requisition for material issue is presented to the store-
he will use the price of the first and then of second and third lot,
keeper
etc.
Suitable :– This method is considered more suitable in times
the prices showwhenthe falling trend. The reason is that higher rate
of
material purchased earlier stands recovered in cost and the
closing
stock is shown at the current
prices.
Merits of FIFO
Method (i) :– It is simple to
(ii) It is a logical method as the materials purchased earlier are
operate.
used in earlier
jobs. is valued at the recent purchase prices, and hence
(iii) Stock
closingstock is valued at the current market
price.method is useful when prices are
(iv) This
falling.
Demerits of FIFO
Method
(i) :–In case of fluctuations in prices of materials,
calculation
becomes
complicated.
(ii) When price fluctuate, there is possibility of clerical
errors.
(iii) This method is not suitable when prices are
rising.
Example :–

Date Purchases
Issues Quantity(kg) Rate per Kg
Quantity(Kgs.)
Dec.3 200 20 ——

223

footer
Dec.4 —- — 100
Dec. 10 —- — 50
Dec. 18 300 18 —-
Dec. 20 —- —- 300
Dec. 28 50 15 —-
Dec. 30 —- — 100

Apply First in First Out


Method.
Solution :–
Stores Ledger
Account

Date Receipts Issues


Balance
Qty. Rate Amount Qty. Rate Amount Qty. Rate
Amount
Dec.3 200 20 4000 ------ ------ ------ 200 20 4000
Dec.4 ----- ------ ------ 100 20 2000 100 20 2000
Dec.10 ----- ------ ------ 50 20 1000 50 20 1000
Dec.18 300 18 5400 ------ ------ ------ 50 20 1000
300 18 5400
Dec.20 ----- ------ ------ 50 20 1000
250 18 4500 50 18 900
Dec.28 50 15 750 ------ ------ ------ 50 18 900
50 15 750
Dec.30 ----- ------ ------ 50 18 900
50 15 750 Nil ------ Nil

(2) Last in First Out (LIFO) :– In last in first out method the
Method materials are issued first and endinglast
received inventory consists of
earlier
acquired materials. This method is also known as replacement
cost
method because the latest purchased goods will correspond to the
current
market prices except that goods were not purchased much earlier.
The
inventories will be valued at oldest lot on hand and these values will
be
quite different from current invoice
Meritsprices. of LIFO
Method
(i) :–Like LIFO method it is simple to
operate.
(ii) This method is useful when prices are
rising.

224

footer
ACCOUNTING FOR
MANAGERS
(iii) In this method production is charged at the recent prices
becausematerials are issued form latest
consignment.
Demerits of LIFO
Method
(i) :–
Like FIFO method, there are chance of occurring clerical
errors.
(ii) Stock are valued at old prices and hence they do not reflect
currentprices
.
(iii) Under this method materials purchased at last are issued
first. Hence materials purchased earlier become
obsolete.stock is valued at cost which doesn’t represent
(iv) Closing
currentconditions.
Example :–

Receipt Issue
sDate Qty(Tons) Rate(Per Ton) Date Qty s
(Tons)
Jan1 100 20 Jan 4 50
Jan 16 300 30 Jan 17 200
Jan 27 50 50 Jan 29 200
Apply Last in First Out
method.
Solution :–
Stores Ledger
Account
Date Receipts Issues
Balance
Qty. Rate Amount Qty. Rate Amount Qty. Rate
Amount
Jan1 100 20 2000 ------ ------ ------ 100 20 2000

Jan2 ----- ------ ------ 50 20 1000 50 20 2000

Jan16 300 30 9000 ------ ------ ------ 50 20 1000


300 30 9000

Jan17 ----- ------ ------ 200 30 6000 50 20 1000


100 30 3000

Jan27 50 50 2500 ------ ------ ------ 50 20 1000


100 30 3000
50 50 2500
----- ------ ------ 50 50 2500 ------ Nil
Jan29 100 30 3000 Nil
50 20 1000

225

footer
(3) Highest in First Out (HIFO) Method :– In this system, the material
with the highest price is issued first. It is based on the assumptions
that
stock should be valued at the lowest possible price. The highest
priced
materials should be issued first, no matter when they are
purchased.
(4) Average Cost Method :– In average cost method of pricing all materials
in stock are so mixed that price based on all lots is formed. Average cost
may
be of two
types:
(a) Simple Average Cost :– In this method the prices of all lots in stock
are averaged and the materials are issued on that average price.
For
example, three lots of materials are in stock and the prices per
unit
these lots are Rs.2, Rs.3, Rs.4 of first, second and third
lots
respectively; then the average price will
be:
2+3+4
Average Price= —————— = Rs.
3 3
Though this is a simple method of pricing materials
but this method does not give good results. The total cost
particularly
is
not observed in this method. The following example will explain
this
point:
10,000 units were purchased @ Rs. 2 per
unit
15,000 units were purchased @ Rs. 3 per
unit
20,000 units were purchased @ Rs. 4 per
unit cost of materials will
The total
be: 10,000 X 2 = 20,000
15,000 X 3 = 45,000
20,000 X 4 = 80,000
Total Cost = 1, 45,000
The simple average price issue in this case is Rs. 3 and
amount total
will become 1,35,000 (45,000X3). The unde r absorbed
amount in this case will be Rs. 10,000. Because of this
weighted
average method is
preferred.
(b) In Weighted
this method the totalAverage
cost of all
the materials
Method :–is divided by the total number of items in stock. The
price
calculated in this way will be used for issue of materials. Taking
the
earlier example the weighed average price will
be:

10000 X 2 + 15000 X 3 + 20000 X 4


Weighted Average Price =
——————————————————— 10000+ 15000+ 20000

226

footer
ACCOUNTING FOR
MANAGERS
1, 45,000
= ————————— = Rs.
3.22 45,000
(5) In Base
this method some Stock
quantity of materials
is Method
assumed :–to be necessary for keeping the concern going. The quantity
is
not issued unless otherwise there is an emergency. This material which
not issued as is kept in stock as a base stock. This method is not
is
an
independent method. It is used alongwith some other methods such
as
FIFO, LIFO, Average Price Method, etc. After maintaining the
base
quantity in stock, the issues are priced at one of the methods
mentioned
above.
(6) The
Standardissue price Price
of materials is
predetermined
Method
or :– in this method. The standard price is based on
estimated
market
conditions, usage rate, storage facilities, etc. The materials are priced
at
standard price irrespective of price paid for various
purchase.
For Example :– The Standard price of raw material is fixed at Rs. 5
per
unit. Two lots of materials of 10000 units and 12,000 units
were
purchased at Rs. 4.90 and Rs. 5.25 per unit. Every issue of material will
be
priced at Rs. 5 per unit, without taking into consideration the prices
at
which these were
purchased.
(7) InMarket
this method the Priceprice charged to
production
Method
are :– incurred on the materials but latest market prices. It
not costs
the latest price charged to production. This method is not generally
reflects
used
because of a number of difficulties. It becomes difficult to select
the
market price because price prevails in different
markets.

227

footer
ACCOUNTING FOR
MANAGERS
MBA 1st Semester
(DDE)

UNIT – IV

Q. Define Budgetary Control. What are the objectives and


Essential
elements of Budgetary
Control?
Ans. Meaning of Budgetary Budgetary control is applied to
Control :– and accounting control by which
of management system
all operations and
outputs
are forecasted so for ahead as possible and actual results when known
as
compared with budget
estimate.
Definition of Budgetary :–
Control
According to Chartered Institute of Management Accountants
(London)
“ Budgetary control is the establishment of the budgets relating to
the
responsibilities of executives to the requirements of a policy and
the
continuous comparison of actual with budgeted results either to so are
by
individual action the objectives of that policy or to provide a firm basis for
its
revision
.
Essential Elements of Budgetary
Control :–
(1) Establishment of Budget for each function and section of the
organization.
(2) Continuous comparison of the actual performance with that of the
budgetso as to know the variations from budget and placing the responsibility
of
executives for failure to achieve the desired results as given in the
budget
(3) Taking suitable remedial action to achieve the desired objective if there
is a variation of the actual performance from the budgeted
performance.
(4) Revision of budgets in the light of changed
circumstances
Objectives of Budgetary :– The main objectives of budgetary
Control
are: control
-
(A) Main
objectives :–
1. Planning
Planning
means:– (a) setting up as objectives (b) setting up
of organization to implement the objectives. Objectives are the targets to
be

228

footer
ACCOUNTING FOR
MANAGERS
achieved in a particular period by the concern within its limited
resources.
The overall planning is represented in the form of master budget which
is
prepared with the help of functional
budget.
2. The Co-ordination
budgetary :– control co-ordinates the
variousactivities of the firm so that the common objective of the firm may
be
successfully achieved. It forces executives to think and think as a group.
It
also helps in co-ordinating the policies, plans and
action.
3. Budgetary
Control :–control makes control possible by
continuous
comparison of actual performance with that of the budget so as report
the
variation from the budget to the management of corrective
action.
(B) Other
Objectives :–
1. UnderDefinite
budgetary control, every department
is Objectives
given a target:– to be achieved. The efforts are made to achieve the
specific
aims
.
2. Budgetary
Policy control helps in the formulation
the Formulation
business policy :– on general or specific matters like materials,
labour,
overheads, sales, production
etc.
3. In budgetary
Cost control, various budgets are prepared
for Control
expenditure:– and then actual are compared with budgeted and
taking
corrective actions for
deviations.
4. UnderMaximumbudgetary control, the resources are
utilizedProfits :– in an organization as each person is aware of his task and
efficiently
the
best way by which it is to be
performed.
5. It aims
Corrective
at taking corrective measures. If there is
a Measures
difference :–between budgets and actual results then corrective actions
are
taken.
6. Economy in The planning of expenditure will
Expenditure
systematic and:– there will be economy inbespending. The resources are
used
to the best
advantage.
7. It Effective
is a very important tool for effective control
because
Control
under it :–
the actual performance is compared with the budgets
and
remedial steps are taken in case of deviation, if
any.
Q. Explain the Different Types of
Budgets.
Ans. Types
Types of Budgetsof :–
are: Budgets
229

footer
Types of
Budgets

According to According to According to


Period
Functions
Flexibility

1. Sales Budget 1. Fixed Budget 1. Long Period Budget


2. Production Budget 2. Flexible Budget 2. Short Period Budget
3. Materials
Budget
4. Labour Budget
5. Overhead Budget.
6. Cash Budget
7. Master
Budget
(A) :–
According to
Functions
1. A sale
Sales
budget is an estimate of expected sales during
a Budget :–
budget period. It is the most important budget and it is called
the
backbone of the enterprise. A sale budget is the starting point on
which
other budgets are
based.
In sale budget expected sales are expressed in quantity as well as in
value.
A sales manager is responsible for preparing sales budget. The
following
factors should be taken into account while preparing sales
budget:
(i) Past sales figures and
facts.
(ii) Availability of Raw
Materials
(iii) Seasonal
Fluctuations
(iv) Plant
Capacity
(v) State of competition in the
market
(vi) Availability of
finance
(vii) Government
policy
(viii) Selling
price
(ix) Development of
Market.
The following informations can be obtained with the help of sale
budget
(i) :– Sales
Target
(ii) Possibility of sales in different
areas.
(iii) What efforts should be made for increasing sales in new
areas.?
(iv) How much amount is required to increase the
sales?
230

footer
ACCOUNTING FOR
MANAGERS
2. Production
Production
budget
Budget
is a:–forecast of production
and cost of production for a budget period. A production manager is
made
responsible for preparing production budget. A production budget
is
prepared on the basis of sales budget. The sales budget presents
demand
while the production budget makes adequate arrangements for
the
fulfillment of this demand. The objective of this budget is to
manufacture
the product at minimum
cost.
3. Material
Materials
budget is prepared for determining
the requirement
Budget :– of raw materials for production. This budget depends
upon
sales and production budget. The materials are purchased as per
the
requirements of production
department.
Total Material Required = Quantity of material required per
unit
X Budgeted Output
Material Cost = Units of Materials required X Rate per unit
of
Raw
4. TheMaterial
Labour
labour Budget
budget :–
required for manufacturing
the product is known as direct labour and the labour which cannot
be
specified with production is called indirect labour. This budget is
useful
for anticipating labour time required for
production.
Labour Cost = Labour Hours X Rate of pay per hour.
5. Overheads
Overhead budget is prepared for the estimation
of Budget :–expenses related to production i.e. indirect material,
indirect
indirect
labour and other indirect
expenses.
6. Cash
Cashbudget
Budgetis :–
a statement of estimates of cash
position
for the budget period. It is a plan of estimated receipts and payments
of
cash for the budget period. It can be prepared for any time period.
Normal
time period of cash budget is half year which is further sub-divided
into
the months. It helps in planning and control of the financial
requirements
of the organization. In cash budget estimated regarding each item of
cash
receipt and payment is made at the time of its
preparation:
(i) Cash-Receipts
Items :–
Ø Cash
Ø Sales
Collection from
Ø Debtors
Interest
Ø Received
Dividend
Ø Receivedof Tax Refund
Amount
Ø Any other income
received
(ii) Cash-Payments
Items Ø
:– Cash purchase of Raw
Materials
231

footer
Ø Payment made to
Ø Creditors
Wages, salaries
Ø paid
Repayment of Bank
Ø Loan
Payment of
Ø Taxes
Any other Expenses
paidis prepared for the business as a
7. A Master
master budget
whole,Budget :– all the budgets for a period into this budget. Thus this
combining
budget
gives the overall budget plan for the guidance of the management.
This
budget is also known as ‘Summary Budget’ or the ‘Finalised Profit Plan’
As
the main objective of budgeting in the profit planning this budget
co-
ordinates all the subsidiary budgets in a summary form and shows
the
final projected results of the
plan.
The following steps are therefore required for preparing a Master
Budget :–
(i) The preparation of sales budget is the basis starting point for
the preparation of the Master
Budget.
(ii) The preparation of the production budget is the next
step.
(iii) Cost of production budget is the third step in preparation of
the Master
Budget.
(iv) The preparation of the cash budget is the next important
step.
(v) The above four steps will be helpful in providing information
for preparing the budgeted or projected income
(vi) Onstatement.
the basis of last year’s balance sheet and the
information
collected by taking above steps, the budgeted or projected
balance
sheet for the business will be prepared. This will be the final step
in
the preparation of a Master
Budget.
(B) :–
According to
Flexibility
1) Fixed
Fixed Budget
Budgetis :–a budget which is desired to
remainunchanged irrespective of the level of activity attained. It does not
change
with the change in level of activity actually
attained.
2) A flexible
Flexiblebudget is a budget designed to change
in Budget :– with the level of activity actually attained. It varies with
accordance
the
level of activity attained. Flexible Budget is desirable in the
following
cases
:(i) Where the business is new or estimation of demand is not
possible.
(ii) Where sales are
unpredictable.
(iii) Where the demand for the product keep changing due to change
in fashion and tastes of
customers.
(iv) When production cannot be estimated due to irregular supply
of necessary materials and
labour.
(C) :–According to
Period
232

footer
ACCOUNTING FOR
MANAGERS
(1) Long
Longperiod
period
budgets
Budgetare
:– those budgets which
are prepared for long
period.
(2) Short
Short
period budgets
Period
are those budgets which
are prepared
Budget :– for short
period.
Q. Write a short note on Zero Base Budgeting
(ZBB).
Ans. Meaning of Zero-base :– The technique of Zero-
Budgeting
Budgeting suggests that an organisation should not onlybase
make decision
about
the proposed new programmes but it should also, from time to time, review
the
appropriateness of the existing programmes. Such review should
be done of such responsibility centres where
particularly there is relatively high
proportion
of discretionary
costs.
Definitio :–
n
“ZBB is a management tool which provides a systematic method for
evaluating
all operations and programmes, current or new, allows for budget
reductions
and expansions in a rational manner and allows re-allocation of sources
low
from to high priority
programmes”.
Process of Zero-Base :–
Budgeting
The process of Zero-base Budgeting involves the following
steps :–
1. Determination of the Objectives of The determination
Budgeting
the :– of budgeting is the first step in the systemofof
objectives
Zero-base Budgeting. The objective may be to effect cost reduction in
introducing
staff
overheads or analyse and drop the projects which do not fit in
the
organizational structure or which are not likely to help in achieving
the
organization’s objectives
etc.
2. Determination of the extent to which the Zero-Base Budgeting is
beto This requires going through the organisation chart
introduced :– pendingor reorganization or programme realignment.
evaluating the
After
studying the organization’s structure, the management can
decide
whether Zero-base Budgeting is to be introduced in all areas
of
organization’s activities or only in a few selected areas on trial
3. basis.
Development of Decision Decision units refer to units
Units :–cost benefit analysis will be done regarding
which to arrive at a decision
they should be allowed to continue or should they be dropped. It may be
whether
afunctional department, a programme, a product-line or a sub-line.
Each
decision unit must be independent of all the other units so that if the
cost
analysis proves unfavorable that unit can be dropped. While
such decision units, the following points should be kept in
selecting
mind:
a) They should be capable of being meaningfully reviewed and
analyzed.
They should, therefore, neither be too low nor too high in
the
organizational
hierarchy
233

footer
b) The managers of these decision units should be capable of
being taking significant decisions keeping on view the scope, direction
and
quality of work to be
4. performed.
Development of decision This is the most important
Packages
involved in:–the ZBB process. After identification of
stepdecision units,
the
manager of each decision unit has to analyse the activities of his
own
decision unit or units. He examines the alternative ways of
his objectives. He does cost benefit analysis and selects the
accomplishing
best
alternative. Then he prepares the decision packages which
effectively
summarize his plans and the resources required to achieve
5. them.
Review and Ranking of Decision The decision packages
Packages
(budget :–
requests) after being developed and formulated are or
submitted
to
next level of responsibility within the organisation for ranking
purposes.
The objective of such ranking is to put the limited resources at the
of the organisation to the best use. The management ranks the
disposal
various
decision packages in order of decreasing benefit or importance to
the
organisation. The preliminary ranking is done by the decision
unit
manager himself who has developed the decision packages. They are
then
sent to the superior officers who once again review and rank the
packages keeping in view the overall objectives of the organisation
decision
in
mind.
Preparation
6. This of in the
is the last stage involved
ZBB Budgets
process. :–
Once the top management has ranked the various
decision
packages keeping in view the cost benefit analysis and the availability
of
funds, a cut-off point is established. All packages which come within
this
cut-off point are accepted and others are rejected. The resources are
allocated to the different decision units and budgets relating to each
then
unit
are
prepared.
Q. Define Standard Costing. Explain variance of standard costing.
Whatare its
Ans. causes?
Standard
Standard Costing
costing is a:– system of cost
accounting
which is designed to find out how much should be the cost of a product
under
the existing conditions. The actual cost can be ascertained only
when
production is undertaken. The pre-determined cost is compared to the
cost
actualand a variance between the two enables the management to
take
necessary corrective
measures.
Standard Costing involves :–
(i) The setting of
Standards
(ii) Ascertaining actual
results.
(iii) Comparing standards and actual costs to determine the
(iv) Investigating the variances taking appropriate action
variances
wherenecessary
.
234

footer
ACCOUNTING FOR
MANAGERS
Advantage of Standard Costing :–
(i) Measuring
efficiency
(ii) Formulation of production & price
(iii)
policy Determination of
variance
(iv) Standard cost may result in a reduction in clerical
work.
(v) Standard costs are useful in decision making and
planning.
Limitation of Standard
Costing
(i) :– Heavy
Cost
(ii) Fixation of Responsibility
difficult.
(iii) Adverse Psychological
Effects
(iv) Unsuitable for Non-Standardized
Products
Varianc :– The object of standard costing is to exercise control and
e
reduction. cost
The deviations between standard cost and actual cost will be
known
as variances. The variances may be favourable and unfavorable. If actual
cost
is less than the standard cost the variances will be favourable and
otherwisebeit
will adverse or
unfavorable.
Classification of It may be classified into various
Variances :– categories:
1. Material Variance 2. Labour
Variance
3. Overheads Cost Variance 4. Sales or Profit
Variance
1. TheMaterial
standard cost of raw material consists of
two elements,
Variance quantity
:– and price. Material variance may be classified
as
follows :–
(i Material Cost Variance This represents the
) between
(MCV) :– the actual cost and the standard costs of
difference
materials.
Material Cost Variance = (SQ X SP) - (AQ X
AP)
SQ = Standard
Quantity
SP = Standard
Price
AQ = Actual
Quantity
AP = Actual
Price
Note :– When there is a difference between standard and actual
output (MCV) standard quantities has to
then to calculate
revised. MATERIAL COST
VARIANCE

Material Price Varianc


Material Usage
Variance e

Material Mix Variance


Material
Yield Variance

235

footer
(ii Material Price Variance This represents the
) (MPV) :– the standard price and actual price
between difference
of materials
consumed.
Material Price Variance = (SP-AP) X
AQ SP = Standard
AQ
Price = Actual
Quantity
AP = Actual
(iii Price
Material Usage Variance This represents the
) (MUV) :–the standard quantity which should
between differences
have been
consumed
and the actual quantity expressed in term of
money.
Material Usage Variance= (SQ-AQ) X
SP SQ = Standard
Quantity
SP = Standard
Price
AQ = Actual
Quantity
(iv) InMaterial
many industries
Mix it happens
Variance
that two
(MMV) :–
or more materials are introduced into a process in a
ratio. This is known as a ‘standard mix’. The cost of the mix
standard
may
therefore differ from standard giving rise to a Materials Mix
Variance.
When there is a difference between the ratios of mix, only then
arises. It is calculated as
MMV
follows:
(a) When the ratios of mix is different but the Total
Standard
Quantities (TSQ) and the Total Actual Quantities (TAQ) are
the
same, in this position the formula of calculating ‘MMV’ will be
under :–
as
Material Mix Variance= (SQ-AQ) X
SP
SQ = Standard
Quantity
SP = Standard
AQ
Price = Actual
Quantity
(b) When the ratio of mix is different and Total Standard
Quantities
(TSQ) and the Total Actual Quantities (TAQ) are also
different,
then standard quantity of each material will be revised. In
position the formula of calculating ‘MMV’ will be as
this
under :–
Material Mix Variance= (RQ-AQ) X
SP RQ = Revised
Quantity
SP = Standard
AQ
Price = Actual
Quantity
Formula for calculating
RQ :– TAQ
RQ = SQ X ————
TSQ

236

footer
ACCOUNTING FOR
MANAGERS
Note :– If the standard quantity is revised the formula of
Material calculating
Usage Variance will be as
under:
Material Usage Variance= (SQ-RQ) X
SP SQ = Standard
Quantity
SP = Standard
Price
RQ = Revised
Quantity Yield
(v) Material Variance Material yield variance
(MYV)
the :– of material usage variance represents
portion which is due to the
difference
between the standard yield specified and the actual yield
obtained.
The actual yield differs from the standard yield due to abnormal
loss.
It is calculated as
under :–
(a) When the Total Standard Quantity and Total Actual
Quantity
are same, then the formula for calculating
MYV :–
MYV = (AY – SY) X SC
OR
MYV = (SL – AL) X SC

AY = Actual
Yield
SY = Standard
Yield
SC = Standard Cost per
unit
SL = Standard Loss
AL = Actual
Loss

Total SC of Standard
SC Mix.
Per unit=
———————————————— SY
(b) When the Total Standard Quantity and Total Actual
Quantity
are not same, then the formula for calculating
MYV :–
MYV = (AY – RSY) X SC
OR
MYV = (RSL – AL) X SC

AY = Actual
Yield
RSY = Revised Standard
Yield
SC = Standard Cost per
unit
Formula for calculating
RSY :–
237

footer
TAQ
RSY = SY X ——––——
TSQ
Relationship between Material The relationship between
Variancesvariances
material :– can be expressed as these
follows
(i):– MCV = MPV +
MUV
(ii) MUV = MMV +
(iii) MCV = MPV + MMV +
MYV
MYV
Causes of Material
Variances :–
(i) Changes in basic price of
materials
(ii) Failures to purchase the quantities anticipated at the time
when standards were
(iii) set.
Failure to secure discount on
purchases.
(iv) Failure to make bulk purchases and incurring more on
(v) Failure to purchase materials at proper
freight.
time.
(vi) Negligence in use of
materials
(vii) More wastage of materials by untrained
workers.
2. There
Labour
may beVariance
two main
:– reasons of the occurrence
of deviations in cost of direct
labour :–
(i) Difference in actual rates and standard rates of labour
and
(ii) The variation in the actual time taken by the workers and
standard
time allowed to them for performing a job. Labour variances
are
classified as
(i) It Labourfollows:-
is the difference
Cost between
Variancethe
standard
(LCV)
labour :–
cost and actual labour cost of the
product.
Labour Cost Variance = (ST X SR) - (AT X
AR)
ST = Standard Time
SR = Standard
Rate
AT = Actual
Time
AR = Actual
Rateknown as LRV.
(ii) This
Labour
is also Rate It is that portion
of Variance
labour cost:–variance which is due to the difference between standard
rate
specified and the actual rate
paid.
Labour Rate Variance = (SR-AR) X
(iii Labour AT Efficiency Variance It is that portion of labour
) (LEV) :– which arises due to the difference between
variance cost the standard
labour
hours specified and the actual labour hours
spent.
Labour Efficiency Variance = (ST-AT) X
SR

238

footer
ACCOUNTING FOR
MANAGERS
(iv) Labour Idle Time It is that portion of labour
variance
Variance :–which may arise due to abnormal wastage of time on account
efficiency
of
strikes, power cut, non-availability of raw-material, breakdown
machinery
of
etc.
Idle Time Variance = Idle hours X Standard
(v) Where
Labour Rate Mix
workers of two orVariance
more than
two types
(LMV) are:–engaged the difference between the standard composition
of
workers and the actual gang of workers is known as ‘Labour
Variance’. It is calculated as
Mix
under:-
(a) When Total Actual Time (TAT) spent and Total Standard Time
(TST)are
same :–
Labour Mix Variance = SR (ST-
AT)Total Actual Time (TAT) spent and Total Standard Time
(b) When
(TST)are not
same:
Labour Mix Variance = SR (RST-
AT)
RST = Revised Standard
FormulaTimefor calculating
RST :– TAT
RST = ST X ————
TST
(vi) Labour Yield Variance It is that portion of labour
(LYV) :–which arises due to the difference efficiency
variance between actual output
worker and standard output of worker specified. It is calculated as
of
follows:
(a) When Total Actual Time (TAT) spent and Total Standard Time
(TST)are
same :– Yield Variance = (Standard output for actual
Labour
mix
– Actual Output) X SC per
unit Total Cost of Standard Mix
S tandard Cost Per Unit=
————————————————Net Standard Output
(vii) When Total Actual Time (TAT) spent and Total Standard Time (TST) are
not same :
–Labour Yield Variance = (Actual Output- Revised
Standard
Output) X Sc per
unit
Formula for calculating Revised Standard
Output: TAT
Revised Standard Output= Standard Output X
———— TST

239

footer
Causes of Labour
Variance :–
1. Change in basic wage
rate.
2. Excessive
3. Use
Overtime.of non-standard material requiring more time to complete
work.
4. Defective machinery, tools and
equipment.
5. Poor working conditions.
6. Inefficiency of
workers.
7. Wrong selection of
workers.
Q. Define Marginal Costing. What are its advantage and
disadvantage?
Ans. Marginal
Marginalcost Costing
is the amount
:– of any given volume
of
output by which aggregate cost is changed if the volume of output is
increased
or decreased by one unit. Marginal Costing is also known as Variable
Costing.
In this technique, only variable costs are charged to operation, process
or
product.
Characteristics of Marginal
Costing
(1) It is :–
a technique of analysis and presentation of costs which
help management in taking many managerial
(2) Alldecisions.
elements of cost-production, administration and selling
and distribution are classified into variable and fixed
components.
(3) Fixed costs are treated as period costs and are charged to Profit & Loss
A/c for the period for which they are
incurred.
(4) The stock of finished goods and work-in-process are valued at
cost only.
marginal
(5) Prices are determined on the basis of marginal costs by
adding‘contribution’ which is the excess of
sales.
Advantages of Marginal :– Advantage of marginal costing are
Costing
following: the
-1. Marginal
Helpfulcosting in
plays aDecision-
significant role
in Making :– decision-making process. This system helps the
managerial
management
in planning, profitability and cost control
etc.
2. Under
Costmarginal costing all the costs are divided into
fixed cost
Control
and:–variable cost. Variable costs of a product are known as
marginal
cost.
3. Marginal
Profit Planning
cost plays:–a vital and important role in
profit planning of an organization. Marginal costing: Break-even point,
P/V
ratio, margin of safety all help in profit
planning.
Disadvantages of Marginal While marginal costing & technique
Costing
said :– a number of merits. But there are some
to have is demerits
also:-

240

footer
ACCOUNTING FOR
MANAGERS
1. The long term price policy, this technique fails to provide
solution.
2. This technique is not suitable for external
reporting.
Q. What are the Managerial Applications of Marginal
Costing?
Ans. Marginal
Introductio
costing technique
:– is a valuable aid
to n
management in taking many managerial decisions. It is a useful tool for
making
policy decisions, profit planning and cost control. The following are some of
the
important managerial problems where marginal costing technique can
be
applied.
1. Pricing
Decisions
2. Profit Planning and Maintaining a Desired Level of
Profit.
3. Make or Buy
Decisions.
4. Selection of a Suitable Sales
Mix
5. Effect of Changes in Sales
Price
6. Alternative Methods of
Production
7. Determination of Optimum Level of
Activity.
8. Evaluation of
Performance
9. Capital Investment
Decisions
1. Fixing
Pricing
of selling prices is one of the most
important
Decisions of
functions :– management. Although prices are generally determined
by
market conditions and other economic factors yet marginal
costing
technique assists the management in the fixation of selling price
under
various circumstances
as:
(i) Pricing under normal
conditions
(ii) During Stiff
Competition
(iii) During trade
depression
(iv) For accepting special bulk
orders
(v) For accepting export orders and exploring new
markets
2. Profit Planning and Maintaining a Desired Level of Margina
Profit :–techniques can be applied for profit planning as well.
costing l
Profit
planning involves the planning of future operations to achieve
maximum
profits or to maintain a desired level of profits. The change in the
sales
price, variable cost and product mix affect the profitability of a
concern.
With the help of marginal costing, the required value of sales
for
maintaining or attaining a desired level of profit may be ascertained
as
follows.
Fixed Cost + Desired
Desired Profit
Sales =
———————————————— P/V Ratio
3. Sometimes
Make a concern
or has Buy
to decide whether
a Decisions :–
certain product should be made in the factory itself or bought from
outside
from a firm which specializes in it. In taking such a ‘make or buy’
decision,
241

footer
the technique of marginal costing is of immense help. While deciding
to
‘make or buy’ a distinction must be made between fixed cost and
variable
cost, and the variable cost of manufacturing it should be compared
with
the price at which this component or product can be bought from
outside.
It is advisable to make than to but if the variable cost of the product
is
lower than the purchase price. But if the purchase price is lower than
the
marginal cost, it would be better to buy than to make
itself.
4. Selection of a Suitable Production/Sales When a
Mix :–
manufacturers more than one product, a problem often arises as to concern
the
product mix or the sales mix which will yield the maximum profits.
In
determining the optimum sales mix, the products which give
the
maximum contribution are to be retained and their production should
be
increased. The production of products which give comparatively
lesser
contribution should be reduced or dropped altogether. Finally
the
optimum sales mix is that which gives the highest
contribution.
Contribution is calculated as
below:
Contribution = Sales – Variable
Cost
5. Effect of Changes in Sales Management is generally
Price
with a:–problem of analyzing the effect of changes
confronted
in sales price upon
the
profitability of the concern. It may be required to reduce the prices
on
account of c ompetition, de pre ssion, expans ion, programme
or
government regulations. The effect of changes in sales prices can be
easily
analyzed with the help of contribution
technique.
6. Alternative Methods of Sometimes the management
Production
to :–
choose from among alternative methods ofhasproduction, e.g.,
machine
work or hand work. The same product may be produced either
by
employing machine No. 1 or Machine No. 2, and the management may
be
confronted with the problem of choosing one among them. In
such
circumstances, technique of marginal costing can be applied and
the
method which gives the highest contribution can be
adopted.
7. Determination of Optimum Level of The technique
Activity :–costing also helps the management in determining the
marginal of
optimum
level of activity. To make such a decision, contribution at different levels
of
activity can be found, and the level of activity which gives the
highest
contribution will be the optimum level. The level of production can
be
raised till the marginal cost does not exceed the selling
price.
8. Evaluation of Evaluation of performance efficiency
Performance
various :–
department, of can also be made with
product lines or markets
the
use of the technique of marginal costing. Sometimes, the
management
may have to decide to discontinue the production of non-
profitable
products or departments so as to maximize the profits. In such cases,
the
242

footer
ACCOUNTING FOR
MANAGERS
contribution of different products, departments or sales divisions can
be
compared and the one which gives the lowest contribution in
comparison
to sales should be
9. discontinued.
Capital Investment The technique of marginal costing
Decision
helps the:–management in taking capital also
investment decisions.
Such
decisions are very crucial for the management and the marginal
costing
technique helps the management in taking capital investment
decisions.
Q. Write a short note
on:- (A) Profit/Volume Ratio (P/V
Ratio)
(B) Angle of
Incidence
(C) Margin of
Safety
(D) Break-even Point and Break-Even
Ans. Chart.
(A) Profit / Volume The profit volume ratio which is also called
Ratio ‘contribution
:– the ratio’ express the relation of
ratio’ or ‘marginal
contribution
to sales and can be expressed as
under :–
Contribution
P/V Ratio = ————————— X
100 Sales
Contribution = Sales – Variable Cost = Fixed Cost +
Profit,can also be expressed
P/V ratio
as :– Sales – Variable Cost S-
Vc
(i) P/V ratio = ————————————— i.e.
——— Sales
S

OR

(ii) Fixed Cost + Profit F +


P P/V ratio = ——————————— i.e.
——— Sales
S

OR

(iii) When two years data are


given :– Change in
profit
P/V ratio= ——————————— X
100 Change in
Sales
243

footer
The P/V ratio, which establishes the relationship between contributions
on
sales, is of vital importance for studying the profitability of operations of
abusiness. It reveals the effect on profit of change in the volume. Higher the
P/V
ratio, more will be the profit and lower the P/V ratio, lesser will be the
profit.
Thus every management aims at increasing the P/V ratio. The ratio can
be
increased by increasing the contribution. This can be done
by:- Ø Increasing the Selling Price per
Ø unit.
Reducing the Variable or Marginal
Cost.
(B) The
Angle
angle of incidence of is the angle between
the sales line and
Incidence :– the total cost line formed at the break-even-point where
the
sales line and the total cost line intersect each other. The angle
of
incidence indicates profit earning capacity of the business. A large angle
of
incidence indicates a high rate of profit and on the other hand, a
small
angle of incidence indicates a low rate of profit. Usually the angle
of
incidence and margin of safety are considered together to indicate
the
soundness of a business. A large angle of incidence with a high margin
of
safety indicates the most favourable position of a
business.
(C) Margin
Margin of safety is ofthe difference between actual
sales and
Safety :– at break-even-point. The excess of actual or budgeted sales
sales
the break-even sales is known as the margin of
over
safety. Margin of Safety = Actual sales- Sales at
For Example :– If B.E.P.
actual sales of a company is Rs. 10,00,000 and the sales
at
break-even-point is Rs. 4,00,000 then margin of safety
is
Margin of safety= 10,00,000 -4,00,000 = 6,00,000
Formulas for calculating Margin of
Safety :– of safety (In units) = Actual sales (in units)- Sales
(1) Margin
at B.E.P.(in units)
(2) Margin of Safety (In RS.) = Actual Sales (In Rs.) – Sales
at B.E.P. (In Rs.)
Profit
(3) Margin of Safety = ———————— X
100 P/V Ratio
Margin of
Safety
(4) Margin of Safety = ——————————— X
100 Actual
Sales
Importance of Margin of :– The size of margin of safety is an
Safety important
indicator of the strength of a business. A business with a greater margin
of
244

footer
ACCOUNTING FOR
MANAGERS
safety is more secured and progressive. The margin of safety can be
improved
by taking the following
steps:-
(i) By increase in the Selling
price.
(ii) By increase the level of
production.
(iii) By reducing the fixed
cost
(iv) By reducing the variable
cost
(v) By substituting unprofitable products with profitable
products
(D) Break-Even-Point and Break-Even The Break-even point
Charts :– as that point of sales volume at which total revenue
be defined may is equal
to
total cost. It is a point of no profits no loss. A business is said to break-
even
when its total sales are equal to its total costs. At this point contribution
is
equal to fixed costs. If a business is producing more than the one
break-
even point there shall be profit to the business organization otherwise
it
would suffer a loss. The detailed study of Break-Even Point is known
as
Break-Even
Analysis.
Formulas for calculating Break-Even- :–
Point

Fixed Cost
(i) Break-even point (In units) =
————————————- Contribution per unit

Fixed Cost
(ii) Break-Even Point (In Rs.) = ———————— X
Sales Contribution

Fixed Cost
(iii) Break-Even Point =
—————————— P/V Ratio

Cash Fixed
(iv) Cash Break-even Point (In Cost
units) =
—————————————— Cash Contribution per
unit

Break – Even Chart :– Break-even chart is a graphic presentation of


marginal
costing. The break-even chart portrays a pictorial view of the
relationships
between costs, volume and profits. The break-even point as indicated in
the
chart is the point at which the total cost line and the total sales line
intersect.
245

footer
Example :– Draw a Break-even –
Chart
Output VC Total Fixed Total Selling
Total (Per unit) VC Cost Cost price
Sales
0 5 0 75000 75000 10 0

5000 5 25000 75000 100000 10 50000

10000 5 50000 75000 125000 10 100000

15000 5 75000 75000 150000 10 150000

20000 5 100000 75000 175000 10 200000

25000 5 125000 75000 200000 10 250000

30000 5 150000 75000 225000 10 300000

Break-Even
Chart :– Y
Total Sales Line
300000
Break-Even Point
275000 Angle of Incidence

250000
Total Cost Line
225000

200000

175000

150000
Margin of Safety

125000

100000
Fixed Cost Line
75000

50000

25000

0 5000 10000 15000 20000 25000 30000 X

Output (In Units)

246

footer
ACCOUNTING FOR
MANAGERS
Past Year Question
Papers

JULY 2007
UNIT–I
1. Discuss the nature, functions and significance of accounting.
2. a) What is Profit and Loss Account ? How does it differ from Trading
Account.
b) Mention any three important adjustments that are made for the
preparation of Trading and Profit & Loss Account.

UNIT–
II
3. What is Fund Flow Statement ? Explain the different sources and
applications of fund.
4. Given the following information, work out debt-equity ratio :

Particulars Amoun
t in
Equity Share Capital Rs.
6,00,000
Preference Share Capital 2,00,000
General Reserve 2,00,000
Profit & Loss Account
P & L A/C Balance
40,00,000
Profit for the year 2,00,000 6,00,000
13% Convertible Debentures 5,00,000
10 year Loan from IDBI 3,00,000
Creditors 1,00,000
Provision for Tax 2,00,000
Bank Overdraft 1,00,000

UNIT–
IIIaccounting information in
5. ‘‘Management accounting is the presentation of
such a way as to assist the management in the creation of policy and in the
day to day operation of the undertaken ? Elucidate the above statement.
6. Distinguish between Management Accounting and Cost Accounting. Explain
various classification of cost in brief.

UNIT–IV
7. What is meant by budgetary control ? Discuss the essentials of a good
budgetary control system.

247

footer
8. The standard cost card reveals the following information :
Labour Rate : Rs. 1/hour
Hours set per unit for production 10 hours
Actual data are given below :
Units produced 500
Hours worked 6,000
Actual Labour Cost 4,800
Calculate Labour variances

JAN 2007

UNIT–I
1. What is a Trial-Balance ? How it is prepared ? Give examples.
2. Define depreciation. Distinguish between Straight Line and Diminishing
Balance Methods. Give examples.

UNIT–
II :
3. From the following details, prepare a Balance Sheet
Current Ratio : 1.75
Liquid Ratio : 1.25
Stock Turnover Ratio : 9 times
Gross Profit Ratio : 25%
Debt Collection Period : 1.5 months
Reserves to Capital : 0.2
Turnover of Fixed Assets : 1.2
Capital Gearing Ratio : 0.6
Fixed Assets to Net worth : 1.25
Sales for the year : Rs. 12,00,000
4. State the significance of preparing a funds flow statement. How ‘Funds from
Operations’ is calculated ?

UNIT–
IIImeet the needs of
5. ‘‘Management Accounting has been evolved to
management.’’ Explain this statement fully.
6. Why is correct valuation of inventory essential ? Explain LIFO & FIFO
methods of inventories invention.

UNIT–IV
7. What is budgetary control ? Discuss various advantages and essentials for
the success of budgetary control.
8. Discuss the different marginal costing applications in managerial decision
making.

JULY 2006
UNIT–I
1. What is the need for providing depreciation ? Discuss with suitable examples
the method of providing depreciation as per Companies Act.

248

footer
ACCOUNTING FOR
MANAGERS
2. The following is the trial balance of Mr. Arthur as on 31 December 2004. st

Dr. Balance Cr.


Balance (Rs.) (Rs.)

Capital a/c — 86,690


Stock on 1 Jan. 2004 46,800 —
st

Sales — 3,89,600
Return Inwards & Outwards 5,800
Purchases 3,21,700 —
Freight & Carriage 18,600 —
Rent and Taxes 5,700 —
Salary & Wages 9,300 —
Debtors & Creditors 24,00 14,800
Bank Loan @ 6% p.a. — 20,000
Bank Interest 900 —
Printing & Advertising 14,600 —
Misc. Income — 250
Cash at Bank 8,000 —
Discount earned — 4,190
Furniture & Fittings 5,000 —
Discount allowed 1,800 —
Misc. Exps. 15,9650 —
Cash in Hand 380 —
Drawing 40,000 —

5,21,330 5,21,330

Following adjustments are to be made :


a) Included amongst the debtors is Rs. 3,000 due from Ankur and included
among the creditors Rs. 1,000 due to him.
b) Personal purchases amounting to Rs. 600 have been included in
purchase day book.
c) Interest on bank loan is to be provided for the whole year.
d) A quarter of the amount of Printing and Advertising is to be carried
forward to the next year.
e) Stock on 31-12-2006 was Rs. 78,600.
Prepare Trading and Profit & Loss Account for the year ended 31-12-2006 and
Balance Sheet as on 31-12-2006.

249

footer
UNIT–
II financial statements of ABC
3. From the following particulars extracted from the
Ltd. Assess the performances over previous year of the company with the help
of relevant ratios and give your comments.

Year I Year II
(Rs.) (Rs.)
Opening Stock 47,000 53,000
Closing Stock 53,000 67,000
Sales less returns 2,52,000 3,65,000
Provision for Bad debts 2,000 3,000
Sundry creditors 32,000 35,000
Purchases 1,80,000 1,90,000
Sundry debtors 42,000 63,000
Cash 10,000 15,000
Bank 15,000 20,000
Bills Receivable 15,000 20,000
Bills payable 29,000 30,000
Marketable Security 8,000 8,000

4. Explain cash flow statement and its salient features. Also explain its uses.

UNIT–
III
5. Define Management Accounting. Discuss the techniques, scope and
limitations of management accounting.
6. Briefly discuss, with examples, the following inventory valuation methods :
a) First in first out b) Last in first out

UNIT–IV
7. The expenses budgeted for the production of 10,000 units in a factory are as
under :
Per unit
(Rs.)
Materials 70
Labour 25
Variable overheads 20
Fixed overheads (Rs. 1,00,000) 5
Variable expenses 10
Selling expenses (10% Fixed) 15
Distribution Expenses 8
Administrative Expenses (Fixed) 6
159

250

footer
ACCOUNTING FOR
MANAGERS
Prepare a budget for production of (i) 8,000 units and (ii) 7,000 units.
8. ‘‘Variance analysis is an integral part of standard cost accounting.’’ Explain.

JAN 2006
UNIT–I
1. Prepare a Trial Balance with ten hypothetical transactions. Also show the
journal entries and ledger postings of those accounts.
2. Define depreciation. Show, with an example, how an asset account is
maintained, if the asset is to be disposed off after three years. You may charge
10% per annum depreciation and can use any of the two method, as per
Company’s Act.

UNIT–
II following ratios :
3. Explain briefly the meaning and usefulness of the
a) Liquidity ratios b) Profitability ratios
4. The following are the comparative balance sheets of XYZ Ltd. :

Liabilities 31-12-2003 31-12-2004


(Rs.) (Rs.)

Share Capital 70,000 74,000


Debentures 12,000 6,000
Trade Creditors 10,360 11,840
Provision for Doubtful debts 700 800
Profits & Loss 10,040 10,560

1,03,200 1,03,200

Assets
Cash 9,000 7,800
Trade Debtors (good) 14,900 17,700
Stock-in-trade 49,200 42,700
Land 20,000 30,000
Goodwill 10,000 5,000

1,030,100 1,03,200

Additional Information :
1. Dividends where paid totalling Rs. 3,500
2. Land was purchased for Rs. 10,000 and amount provided for the
amortization of goodwill totalled Rs. 5,000.
3. Debenture loan was repaid Rs. 6,000.
You are required to prepare Cash Flow Statement.

251

footer
UNIT–
III
5. Briefly explain what are the functions of management accountant.
6. ‘‘Standard costing is an essential tool of management.’’ Comment on this
statement.

UNIT–IV
7. What do you mean by Marginal Costing ? Discuss in brief its application in
managerial decision making.
8. ‘‘Variance analysis is an integral part of standard cost accounting. Explain.

JULY 2005

UNIT–I
1. What are the accounting concepts and conventions ? Explain any three of the
following :
i) Dual Aspect Concept
ii) Cost Concept
iii) Convention of conservation
iv) Business Entity Concept
2. a) Write a short note on Trial Balance.
b) Write a short note on Balance Sheet.
c) Write a short note on Depreciation.

UNIT–
II
3. ‘‘A cash flow statement is required to explain change in cash account
balances between balance sheet date.’’ Explain the statement.
4. From the following particulars, prepare the balance sheet of the form
concerned:
Stock velocity = 6
Capital turnover ratio = 2
Fixed assets turnover ratio = 4
Gross profit = 20%
Debt collection period = 2 months
Creditors payment period = 73 days
The gross profit was Rs. 60,000
Closing stock was Rs. 5,000 in excess of the Opening stock.

UNIT-
III of ABC Ltd. for the year
5. Given below are the changes in Account Balance
ending 31 March, 2002.
st

Rs.
Cash (+) 96,000
Debtors (–) 16,000
Provision for D/D (–) 400
Stock (–) 30,000

252

footer
ACCOUNTING FOR
MANAGERS
Plant (+) 50,000
Accumulated Depreciation (+) 20,000
Bill payable (–) 10,000
Outstanding Expenses (+) 800
Rajeev’s Capital (+) 89,600

Debtors Rs. 2,000 were written-off as uncollectable. Plant costling Rs. 17,500
was sold for Rs. 7,500 resulting a loss of Rs. 1,000. Net income after charging
the loss on plant amounted to Rs. 1,30,000. Prepare Fund-flow Statement.
6. Define inventory. Why proper valuation of inventory is important ? Also
explain the LIFO and FIFO method of inventory valuation.

UNIT–IV
7. Define budget and budgetary control. Also discuss the advantages and
limitations of budgetary control system.
8. Due to industrial depression, a plant is running at present, at 50% of its
capacity. The following details are available :
Cost of production per unit
Direct Material Rs. 2
Direct Labour 1
Variable Overheads 3
Fixed Overhead 2
8
Production per month 20,000 units
Total cost or production Rs. 1,60,000
Sales Price Rs. 1,40,000
Loss 20,000

An exporter offers to buy 5,000 units per month at the rate of Rs. 6.50 per unit
and the company hesitates to accept the offer for fear of increasing its already
large operating losses. Advise whether offer should be accepted or not.

253

footer
WORKSHEET

254

footer
ACCOUNTING FOR
MANAGERS
WORKSHEET

255

footer
WORKSHEET

256

footer

Das könnte Ihnen auch gefallen